2021 ITE

Réussis tes devoirs et examens dès maintenant avec Quizwiz!

A previously healthy 38-year-old female with a normal BMI has a few warts at the base of her great toe on the plantar surface. She noticed them a few years after she started to run regularly for exercise but has not tried any at-home treatments. They are minimally raised and rarely painful but occasionally cause irritation and she would like to get rid of them. Which one of the following would be the most appropriate treatment? A) Over-the-counter salicylic acid B) Candida injections C) Application of duct tape D) Manual paring and extraction E) Laser treatment

A A Cochrane review reports good evidence that salicylic acid is effective for the treatment of plantar warts. Candida injections may be indicated for warts that are difficult to treat, but they are not considered first-line treatments. The application of duct tape has not been shown to be more effective than placebo.

A 58-year-old male sees you for evaluation of left ankle pain after he slipped on some ice in his driveway last night. He felt immediate pain over the lateral ankle, which started swelling over the next hour. He elevated his foot, applied ice, and took ibuprofen. This morning the ankle remains swollen and also appears bruised. He is able to walk on it with some pain. On examination you note typical findings of an ankle sprain. He asks you for medication to manage his pain. Which one of the following medications has the best evidence for providing pain relief while also minimizing side effects? A) Topical diclofenac gel (Pennsaid) B) Topical menthol gel C) Oral hydrocodone/acetaminophen (Lortab) D) Oral ibuprofen E) Oral tramadol

A AAFP and ACP recs topical NSAIDs as first line therapy for non low back pain related MSK injuries

A sexually active 45-year-old female who has been using oral contraceptives for years without experiencing any problems asks how much longer she should continue contraception. She is happy with her current low-dose estrogen combined oral contraceptive and would like to continue this if possible. She asks if the health risks are high enough to warrant a change to another option. She does not smoke and is in excellent health. Which one of the following would be most appropriate for this patient? A) Continue her current oral contraceptive B) Discontinue contraception C) Obtain FSH and LH levels today and discontinue contraception only if the results confirm infertility D) Replace the oral contraceptive with a barrier method

A ACOG recommend that women continue contraceptive use until menopause or age 50-55 years

An asymptomatic 55-year-old female sees you for the first time. She asks for advice about laboratory results from a health maintenance examination performed last month by her previous physician. These results included a TSH level of 0.2 U/mL (N 0.4-4.2). Other laboratory results, including free T4 and free T3 levels, were normal. Her past medical history and a physical examination today, including a thyroid examination, are unremarkable. Which one of the following would you recommend at this time? A) Observation only B) A thyroglobulin level C) Thyroid antibody studies D) Thyroid ultrasonography E) A thyroid uptake scan with radioactive iodine

A ATA recs observation for asymptomatic patients with mildly low TSH, so further evaluation is not recommended at this point.

A 73-year-old female sees you because of a 2-month history of the gradual onset of bilateral swelling of several metacarpophalangeal joints with associated joint stiffness, which lasts for about 1 hour each morning. Her past medical history and an examination are otherwise unremarkable. Elevation of which one of the following would provide the most support for a specific diagnosis of her new arthritis symptoms? A) Anti-citrullinated protein antibody B) C-reactive protein C) Erythrocyte sedimentation rate D) Leukocyte count and differential E) Rheumatoid factor

A Anti-citrullinated protein antibody is >95% specific for RA when significantly elevated. Acute phase reactants are very nonspecific and may be positive due to infection, other autoimmune conditions, age, or obesity. Leukocyte counts may be elevated because of infection, cancer, smoking, and other conditions. Rheumatoid factor is also nonspecific for RA and may be positive due to cancer, infection, and other autoimmune conditions

In patients diagnosed with COPD, testing should be considered for which one of the following underlying conditions? A) alpha 1-Antitrypsin deficiency B) Cystic fibrosis C) Hemochromatosis D) Williams syndrome E) Wilson's disease

A Clinicians should consider measuring the 1-antitrypsin level in all symptomatic COPD patients with fixed airflow obstruction, particularly with a COPD onset as early as the fifth decade of life; a family history of 1-antitrypsin deficiency; and emphysema, bronchiectasis, liver disease, or panniculitis in the absence of a recognized risk factor

A 2-year-old male with a barking cough is brought to the urgent care clinic by his parents. He is noted to have stridor when agitated and mild retractions. He has a normal level of consciousness, good air entry, and no evidence of cyanosis. Which one of the following treatment modalities would be most appropriate? A) Dexamethasone B) Heliox C) Humidified air inhalation D) Nebulized epinephrine E) Oxygen

A Corticosteroids should be used in the treatment of croup regardless of the degree of severity. Dexamethasone is preferred because it can be given in a single dose and administered either orally, parentally, or intravenously

According to the Ottawa knee rule, a radiograph would be indicated for a patient presenting with an acute knee injury if the examination reveals tenderness to palpation over the A) fibular head B) lateral joint line C) medial joint line D) tibial tubercle E) upper medial aspect of the tibia

A Criteria for imaging according to the Ottawa knee rule include any of the following: age >55, isolated tenderness of the patella, tenderness of the fibular head, inability to flex the knee to 90°, and inability to bear weight for four steps both immediately after the injury and at the time of the examination.

. An 85-year-old male with hypertension and coronary artery disease comes to your office for a routine wellness visit. He is accompanied by his wife, who notes that the patient's memory has been worsening over the last few years. His current medications include carvedilol (Coreg), losartan (Cozaar), hydrochlorothiazide, and aspirin. A brief screening test is positive for cognitive impairment. The Saint Louis University Mental Status (SLUMS) examination places the patient in the dementia category. You order further testing, including a TSH level, a CBC, a comprehensive metabolic panel, and a vitamin B12 level. Which one of the following should also be included in the workup? A) The Geriatric Depression Scale B) Apolipoprotein E epsilon 4 allele testing C) An EEG D) A PET scan E) Cerebrospinal fluid analysis

A Depression in the elderly can cause symptoms similar to those of dementia. Also, many patients with dementia concurrently have depression. It is recommended that depression be treated first if found (SOR C). If cognitive symptoms improve with depression treatment, pseudodementia is diagnosed.

A 14-year-old female is brought to your office after a school screening program identified possible scoliosis. She plays basketball at school and has no history of recent injuries. She is feeling well today and a review of systems is negative. A physical examination reveals an elevated right rib on the forward bend test. Radiography demonstrates a Cobb angle of 15°. Which one of the following would be most appropriate at this point? A) Observation only B) Suspension of sports participation C) Bracing D) Physical therapy E) Surgical evaluation

A Despite a lack of consensus between major health care organizations on the benefit of screening for scoliosis, more than half of states require or recommend school-based screening programs. Adolescent idiopathic scoliosis is generally defined as a lateral curvature of the spine or Cobb angle 10°. Cases with a Cobb angle <20° can generally be managed with observation.

An otherwise healthy 46-year-old female presents with a 10-day history of recurring pain in the right cheek and gums. She says the pain feels like electric shocks lasting a few seconds and recurring "hundreds of times a day." She says that smiling and brushing her teeth can trigger the pain. She does not have a history of recent dental work, trauma, fever, or myalgia. She feels well aside from the facial pain. She has tried over-the-counter analgesics without relief. Her vital signs and an HEENT examination are normal. There is no pain with dental percussion, and the skin and mucous membranes of the nose and mouth are unremarkable. Which one of the following would be the most appropriate treatment for this patient's condition? A) Carbamazepine (Tegretol), 200 mg twice daily B) Prednisone, 40 mg daily C) Sumatriptan (Imitrex), 6 mg subcutaneously D) Valacyclovir (Valtrex), 1000 mg three times daily E) High-flow oxygen for 20 minutes

A First line tx for trigeminal neuralgia is carbamazepine or oxcarbazepine. Other suggested treatments include lamotrigine, baclofen, and surgical treatments, including microvascular decompression. There is a paucity of good evidence for treatments other than carbamazepine.v

A 36-year-old female sees you for a routine health maintenance visit. She reports worsening hair growth on her chin and abdomen over the last few years. The excessive hair growth first appeared in her late teens and she has been dissatisfied with the cosmetic results of various hair removal methods. She is generally healthy aside from a BMI of 31 kg/m2 . She does not take any medications, is a nonsmoker, and has had a bilateral tubal ligation. Her menses are regular. A complete physical examination is consistent with some terminal hairs in the distribution she described and is otherwise unremarkable. Laboratory results are normal, including fasting lipids, a comprehensive metabolic panel, a hemoglobin A1c, and a TSH level. Which one of the following would be the recommended first-line treatment for this patient's condition? A) Drospirenone/ethinyl estradiol (Yasmin) B) Flutamide C) Leuprolide (Eligard) D) Metformin (Glucophage) E) Spironolactone (Aldactone)

A First-line therapy for hirsutism in women who do not desire pregnancy and for whom cosmetic treatments are not effective is combined oral contraceptives (SOR B), which decrease androgen production in the ovaries by decreasing LH levels.

A 55-year-old female with type 2 diabetes sees you because of early satiety, nausea, vomiting, bloating, and postprandial fullness that is sometimes accompanied by upper abdominal pain. Since these symptoms have developed she has also noted increasing difficulty with blood glucose control. Which one of the following would be the best study for confirming the most likely diagnosis? A) Gastric emptying scintigraphy with a solid meal B) Hepatobiliary scintigraphy (HIDA) C) An upper gastrointestinal series with small-bowel follow-through D) Abdominal ultrasonography E) Abdominal CT

A Gastroparesis is a complication of diabetes mellitus, and presents with nausea, vomiting, early satiety, bloating, postprandial fullness, and/or upper abdominal pain. Gastric emptying scintigraphy with a solid meal is the first-line study for confirming the diagnosis.

A 30-year-old male is diagnosed with hereditary hemochromatosis. Periodic therapeutic phlebotomy may be appropriate to prevent A) chronic liver disease B) chronic renal disease C) encephalopathy D) myelofibrosis E) Wilson disease

A Hereditary hemochromatosis is a common inherited disorder of iron metabolism. Iron deposits in the liver may lead to chronic liver disease and hepatocellular cancer.

A 13-year-old male is brought to your office for evaluation of back pain. Plain radiography would be indicated at this time if the patient has A) pain that awakens him at night B) pain that is localized to the midthoracic spine C) pain that is increased with flexion D) intermittent pain that has persisted for 2 weeks E) a recent history of an upper respiratory infection

A Imaging in children/adolescents with back pain can be delayed unless there are abnormal neuro findings, pain that awakens the patient at night, or pain that radiates or persists for more than 4 weeks.

A 50-year-old male has an acute upper respiratory infection and cough that has improved but has not resolved completely. He presents to your office today with a 2-day history of chest pain that began gradually. The pain is worse when he is supine, takes deep breaths, or coughs, and he says the pain is relieved when he leans forward while sitting. He is afebrile and his vital signs are normal. An EKG confirms your impression of acute pericarditis, a troponin level is normal, and he does not appear acutely ill. You treat him as an outpatient with ibuprofen, 600 mg three times daily, and omeprazole (Prilosec) for gastrointestinal protection. The patient returns for follow-up 7 days later and tells you that the pain is somewhat improved but still present. He remains afebrile and his other vital signs are normal. On examination he still has a pericardial friction rub but no gallops or murmurs, and his lungs are clear. Which one of the following would be most appropriate at this time? A) Continue his current medications and add colchicine (Colcrys) B) Continue his current medications and add prednisone C) Discontinue his current medications and start aspirin D) Discontinue his current medications and start indomethacin E) Admit him to the hospital and consult a cardiologist

A In a patient with acute pericarditis, after determining that the patient is not at high risk for complications, does not have acute myocardial injury, and is an appropriate candidate for outpatient treatment, there are several options for treatment. Any of the NSAIDs alone are effective in many patients, but some patients do not respond sufficiently, so the addition of colchicine would be the treatment of choice. Colchicine alone is also an appropriate initial treatment, but in case of insufficient response to NSAIDs

An 84-year-old female presents with dryness and irritation in her eyes. Her optometrist recently diagnosed her with dry eye. Which one of the medications in this patient's current regimen is most likely causing her dry eye? A) Amitriptyline B) Empagliflozin (Jardiance) C) Levothyroxine (Synthroid) D) Liraglutide (Victoza) E) Metformin (Glucophage)

A Many systemic drugs have been reported to trigger dry eye, including diuretic agents, -blockers, other antihypertensive agents such as candesartan, antihistamines, decongestants, medications for Parkinson's disease, antidepressant agents such as amitriptyline, anxiolytic agents, antispasmodic agents, anticonvulsant agents, gastric protection agents, oral contraceptives, and some herbal supplements

A 19-year-old female presents with a 4-year history of intermittent facial acne. She tells you that her acne has never completely resolved, and it worsens during her menstrual period. She has tried various over-the-counter facial cleansers although she does not recall what they contained. On examination she has scattered open and closed comedones, and pustules on her forehead and around her mouth. Which one of the following treatments would you recommend? A) Topical adapalene (Differin) B) Topical clindamycin (Cleocin T) C) Oral doxycycline D) Oral isotretinoin (Absorica)

A Mild inflammatory acne = comedones + pustules. No extensive skin involvement. Topical antibiotics should not be used as monotherapy. Oral antibiotics - use with moderate/severe acne that has failed initial treatment Oral isotretinoin is reserved for the treatment of severe nodular acne

A 39-year-old male with no significant past medical history presents to your office to establish care. He does not take any medications. A review of systems is negative. He has a BMI of 22 kg/m2 and a blood pressure of 141/82 mm Hg. Which one of the following would be the most appropriate next step? A) Ambulatory blood pressure monitoring B) Recommending a weight loss of 5 lb C) Hydrochlorothiazide, 25 mg daily D) Lisinopril (Prinivil, Zestril), 10 mg daily E) Metoprolol tartrate (Lopressor), 25 mg twice daily

A Of the options listed, ambulatory blood pressure monitoring should be used to confirm the diagnosis of hypertension and screen for potential "white coat" hypertension, given the single elevated reading in this new patient.

A 5-year-old female has acute, severe bacterial sinusitis. Which one of the following would be most appropriate for this patient? A) Amoxicillin/clavulanate (Augmentin) B) Cephalexin C) Clarithromycin (Biaxin) D) Doxycycline E) Levofloxacin

A Patients with acute bacterial sinusitis who do not improve while taking the usual dose of amoxicillin, who have recently been treated with an antimicrobial (within the past 90 days), who have an illness that is moderate or more severe, or who attend day care should be treated with high-dose amoxicillin/clavulanate in two divided doses. Alternate therapies include cefdinir, cefuroxime, or cefpodoxime.

Which one of the following would be most important for reducing the risk of recurrence after a single mild episode of diverticulitis? A) Increasing dietary fiber B) Permanently eliminating nuts, seeds, and corn from the diet C) Treating the initial episode with appropriate broad-spectrum antibiotics D) CT imaging early in the episode to detect complications E) Colonoscopy 4-6 weeks after the episode has resolved

A Risk factors for diverticulitis include low dietary fiber, a sedentary lifestyle, obesity, and smoking. Avoidance of nuts, seeds, and corn has not been shown to decrease risk for diverticular disease, including diverticulitis.

A 60-year-old male who has type 2 diabetes comes to your office with an acute onset of fever, chills, and malaise. He says that he is feeling progressively worse. His temperature is 40°C (104°F). An examination reveals redness, tenderness, and swelling of the penis, scrotum, and perineal area. Which one of the following medications is most likely to cause this condition? A) Dapagliflozin (Farxiga) B) Exenatide (Byetta) C) Insulin glargine (Lantus) D) Pioglitazone (Actos) E) Sitagliptin (Januvia)

A SGLT2 inhibitors (canagliflozin, dapagliflozin, empagliflozin, and ertugliflozin) are associated with a higher rate of genitourinary infections, including necrotizing fasciitis of the perineum (Fournier's gangrene).

13. Which one of the following represents a CONTRAINDICATION to statin use? A) Current pregnancy B) Chronic hepatitis C infection C) End-stage renal disease D) Myositis associated with a creatine kinase level five times the upper limit of normal E) Transaminitis due to nonalcoholic steatohepatitis

A Statins have been associated with fetal anomalies and are contraindicated during pregnancy and not recommended during breastfeeding.

A 72-year-old female with a history of type 2 diabetes and hypertension presents to your clinic because of fatigue and depression for the last 5-6 months. She has gained about 7 kg (15 lb) and now has a BMI of 32 kg/m2 . A physical examination is otherwise unremarkable. Laboratory studies reveal a TSH level of 8.2 U/mL (N 0.4-4.0). A repeat test 1 month later shows a TSH level of 7.4 U/mL and a free T4 level of 1.6 ng/dL (N 0.8-2.8). Treatment of this patient with L-thyroxine A) has no proven benefit B) can increase grip strength C) can increase her energy level D) can help improve depression symptoms E) can reduce her BMI

A Subclinical hypothyroidism (SCH) is defined as an elevation in TSH level with a normal free T4 level. It is relatively common in adults over the age of 65, w

The American Thoracic Society/Infectious Diseases Society of America guidelines recommend which one of the following for the diagnosis and initial management of non-severe community-acquired pneumonia in adults? A) Use of a validated clinical prediction rule to determine the need for hospitalization B) Urine antigen testing for Legionella C) Blood and sputum cultures to guide therapy D) Procalcitonin to determine the need for antibacterial therapy E) Coverage for methicillin-resistant Staphylococcus aureus (MRSA)

A The ATS and the IDSA guideline recommends use of a validated clinical prediction rule, preferable the Pneumonia Severity Index (PSI) to determine the need for hospitalization in adults diagnosed with community-acquired pneumonia.

You are covering a weekend shift in the local intensive-care unit. When providing care for a very ill adult patient with hypoproliferative thrombocytopenia who is not currently bleeding, prophylactic platelet transfusion should be considered if the platelet count is below a threshold of A) 10,000/ L B) 20,000/ L C) 25,000/ L D) 50,000/ L E) 100,000/ L

A The threshold for transfusing platelets to prevent spontaneous bleeding in the setting of hypoproliferative thrombocytopenia in most adults is <10,000/ L (SOR A). A platelet count <20,000/ L is the threshold for use of elective central venous catheter placement. For elective diagnostic lumbar puncture, major elective non-neuraxial surgery, and interventional procedures, the threshold is a platelet count <50,000/ L. For neuraxial surgery a threshold <100,000/ L is recommended.

The administrator in your practice recently attended a quality improvement conference. He would like to start a clinic-wide program to focus on lead screening in asymptomatic children 5 years of age and younger. After reviewing U.S. Preventive Services Task Force guidelines, which one of the following should you tell the practice administrator? A) There is insufficient evidence to recommend for or against lead screening in children 5 years of age and younger B) All children 5 years of age and younger should undergo lead screening C) All children who live in housing built before 1978 should undergo lead screening D) There are accurate and reliable screening questionnaires to guide lead screening

A The U.S. Preventive Services Task Force (USPSTF) has found adequate evidence that questionnaires and other clinical prediction tools to identify asymptomatic children with elevated blood lead levels are inaccurate. The USPSTF went on to conclude that the current evidence is insufficient to assess the balance of benefits and harms of screening for elevated blood lead levels in asymptomatic children 5 years of age and younger. Although children living in older housing with lead-based paint are at higher risk of elevated blood lead levels than those living in housing built after 1978, the USPSTF does not recommend routine screening in asymptomatic children based on this risk factor.

A 62-year-old female sees you for a routine health maintenance visit and asks for your advice regarding vitamin D supplementation. She is healthy and active, and jogs 1-2 hours three times weekly. Which one of the following would be the most appropriate advice regarding vitamin D supplementation in this patient? A) It is not recommended because she is asymptomatic B) It will reduce the risk of certain cancers C) It will reduce the risk of depression D) It will reduce the risk of diabetes mellitus E) It will reduce the risk of fractures

A The U.S. Preventive Services Task Force and the American Academy of Family Physicians concluded that evidence is insufficient for vitamin D testing and for vitamin D supplementation in asymptomatic adults. Vitamin D supplementation does not reduce the risk of cancer, depression, diabetes mellitus, or fractures.

A healthy 50-year-old male with no significant past medical history comes to your office for a health maintenance examination. He does not take any medications and does not smoke cigarettes or drink alcohol. He tells you that a friend who is his age recently had an acute myocardial infarction and he would like testing to help decrease his own risk. On examination he has a BMI of 30 kg/m2 . In addition to checking his lipid profile and hemoglobin A1c, evidence supports which one of the following to assess his risk of cardiovascular disease? A) No further testing B) An ankle-brachial index C) A high-sensitivity C-reactive protein level D) A coronary artery calcium score E) A PLAC test for lipoprotein-associated phospholipase A2 activity

A The U.S. Preventive Services Task Force recently concluded that there is insufficient evidence to assess the risk-benefit ratio of screening asymptomatic adults for cardiovascular disease risk by checking an ankle-brachial index, a high-sensitivity C-reactive protein level, or a coronary artery calcium scor

A 13-year-old baseball player who is right-hand dominant is brought to your office because of a 3-week history of pain in the right shoulder. He recalls no specific injury but has been pitching at least weekly for the past 3 months. He has moderate tenderness about the anterior and lateral deltoid. In addition to radiographic studies, the best initial management would include A) complete rest from throwing activities B) a sling and swathe C) sugar-tong splints both proximal and distal to the elbow D) physical therapy

A The insidious onset of this patient's pain without known injury and the lack of spontaneous resolution strongly suggest an overuse injury.

A 58-year-old male with a history of a neurogenic bladder comes to your office as a new patient. He recently elected to have placement of a chronic indwelling urethral catheter rather than performing intermittent catheterization at home, and he asks how to reduce his risk of urinary tract infections (UTIs). His last UTI was approximately 1 year ago and required intravenous antibiotics. Which one of the following is most effective for preventing UTIs in patients with chronic indwelling urethral catheters? A) Routine daily hygiene of the meatal surface with soap and water B) Daily periurethral cleaning with iodine C) Daily oral antibiotics based on prior urine culture sensitivities D) Routine instillation of an antimicrobial solution into the drainage bag E) Regularly scheduled catheter exchanges at fixed intervals

A The most important measure to prevent CAUTIs is routine cleaning of the meatal surface with soap and water while bathing or showering.

A 70-year-old male is brought to your office by a family member. The patient is concerned about a tremor, which is most noticeable at rest and seems to get better with voluntary movements. On examination his vital signs are unremarkable. He blinks infrequently during the examination, his face seems to be relatively immobile, and he feels rigid. He has some difficulty rising from a chair and he walks with small, shuffling steps. The remainder of the neurologic examination is normal. You note some seborrhea of the scalp. A recent comprehensive laboratory evaluation is unremarkable. Which one of the following is necessary to confirm the diagnosis? A) No diagnostic testing B) CT of the brain C) MRI of the brain D) EEG E) A lumbar puncture

A The prevalence of Parkinson's disease increases with age and shows a slight predominance toward males. Bradykinesia is a key diagnostic criterion. Muscular rigidity, resting tremor, and postural instability are other symptoms. Nonmotor symptoms such as depression, anxiety, fatigue, and insomnia are also common. Parkinson's disease is a clinical diagnosis and seldom requires testing.

A 4-month-old female is brought to your office by her mother for a well child visit. The mother tells you about some red patches on the child's cheeks and legs that do not seem to bother the infant. She says that the patches sometimes appear very irritated and improve with occasional lotion use but keep coming back. The mother has not noticed any signs of illness. An examination reveals a well appearing infant with normal growth, development, and vital signs. You note slightly rough erythematous patches on both cheeks and her chin, as well as on her thighs. Which one of the following would be most appropriate at this time? A) Twice-daily application of a fragrance-free moisturizer with a high lipid-to-water ratio B) Twice-daily application of a low-potency topical corticosteroid C) Application of a topical calcineurin inhibitor for the facial lesions and a low-potency topical corticosteroid for the other areas D) Allergy testing E) Referral to a dermatologist

A This infant has skin findings that are consistent with atopic dermatitis. The first-line treatment is liberal use of fragrance-free emollients, at least 1-2 times per day. The use of topical calcineurin is not indicated in children <2 years of age. Allergy testing not recommended in routine eval of atopic derm.

A 45-year-old left hand-dominant female presents to your office with a lump on her hand. She first noticed the lump 2 weeks ago and thinks it has gotten bigger. She does not recall any injury. She has not had any numbness, weakness, or tingling. She has minimal discomfort when she presses on the lump, and it does not affect her activity. On examination her left wrist is neurovascularly intact. You note the volar wrist mass shown below. Which one of the following management options would you recommend? A) Re-examination if she develops numbness, weakness, or increased pain B) Immobilization of the wrist for 6 weeks and then re-examination C) Aspiration of the lesion D) Aspiration and injection of the lesion with a corticosteroid E) Referral for excision of the lesion

A This patient has a ganglion cyst, which is common and resolves spontaneously in 50% of cases, and watchful waiting would be most appropriate at this time.

A 2-year-old female is brought to the urgent care clinic because of a fever. On examination she has a temperature of 39.7°C (103.5°F). Within a short period of time while at the clinic she develops a barking cough and respiratory distress, and you note rapid deterioration of her condition. Which one of the following is the most likely diagnosis? A) Bacterial tracheitis B) Epiglottitis C) Foreign body aspiration D) Peritonsillar abscess

A This patient has bacterial tracheitis, which includes a high fever, barking cough, respiratory distress, and rapid deterioration. Epiglottitis has an acute onset of dysphagia, drooling, and high fever, along with anxiety and a muffled cough, and typically occurs in children 3-10 years of age. Foreign body aspiration is associated with an acute onset of choking and drooling. A peritonsillar abscess would cause a sore throat, fever, and "hot potato" voice.

A 2-year-old female is brought to your office by her mother because of a cough and fever. The mother also tells you that the child has had a reduced appetite but she is drinking fluids normally. The child was born at term and has previously been healthy. On examination the child appears alert and happy. She has a temperature of 37.2°C (99.0°F), a pulse rate of 100 beats/min, a respiratory rate of 30/min, and an oxygen saturation of 98% on room air. An HEENT examination reveals clear rhinorrhea. Auscultation of the lungs reveals mild expiratory wheezing throughout with no crackles, and you note no signs of respiratory distress such as retractions or use of accessory muscles of respiration. Which one of the following would be the most appropriate next step? A) Reassurance only B) A nasal swab for respiratory syncytial virus C) A chest radiograph D) Nebulized albuterol E) Oral amoxicillin

A This patient has symptoms typical for respiratory syncytial virus (RSV) bronchiolitis. Since the patient shows no signs of distress and is well hydrated, no specific treatment is necessary.

A 38-year-old female presents with a 1-week history of pain in her left heel. The pain started abruptly while she was playing tennis and she was unable to finish the game. She has been applying ice, taking ibuprofen, and wearing an ankle brace without improvement. She walks with a limp. An examination is significant for localized tenderness over the left posterior leg approximately 3 cm above the calcaneus. The Thompson test (squeezing the calf) is significant for the absence of plantar flexion of the left foot. Her strength is decreased with plantar flexion of the left ankle. There is normal passive range of motion, as well as normal pulses and sensation in the left foot. Which one of the following is the most likely cause of her pain? A) Achilles tendon rupture B) Plantar fasciitis C) Proximal fifth metatarsal fracture D) Stress fracture of the fibula E) Syndesmosis injury (high ankle sprain)

A This patient most likely has an Achilles tendon rupture based on the history and examination, with pain localized to the posterior leg 2-6 cm above the calcaneus. The Thompson test is positive when there is no plantar flexion of the foot with squeezing the calf, due to a disruption of the Achilles tendon.

A 79-year-old male presents to your primary care clinic as a new patient. He is unaccompanied and tells you that he has no acute concerns and takes no regular medications, but just "needed a doctor." He is a retired lawyer and states that he lives alone. He has no significant past medical history but you are able to access his electronic medical records and note that he has a history of diabetes mellitus and hypertension, and has been to the local emergency department (ED) twice in the last 4 months for headaches. A workup from the ED was notable only for moderately elevated blood pressure. When asked, he does not appear to recall these visits. His vital signs are notable only for a blood pressure of 165/90 mm Hg. A physical examination reveals a thin but well appearing older adult who is mildly disheveled in appearance, with no other abnormalities. Which one of the following would be the most appropriate next step in the care of this patient? A) A cognitive assessment B) Immediate referral to adult protective services C) Immediate referral to a neurologist D) Immediate referral to a psychiatrist

A This patient's history and the physical examination are concerning for features of self-neglect. At this point in the evaluation, the etiology of this possible self-neglect remains unclear. A formal cognitive eval is recommended to eval for objective findings of impairment that may match the subjective concerns. Also consider depression screening.

A 57-year-old male recently diagnosed with acute lymphoblastic leukemia presents to the emergency department with intractable nausea, vomiting, and myalgias. His first chemotherapy infusion was administered earlier in the day. Which one of the following electrolyte disturbances would be consistent with tumor lysis syndrome? A) Hypocalcemia B) Hypokalemia C) Hyponatremia D) Hypophosphatemia E) Hypouricemia

A Tumor lysis syndrome is a common complication of chemotherapy in hematologic malignancies, such as acute leukemia. Homeostasis is overwhelmed with phosphorus, potassium, calcium, and uric acid released into the bloodstream due to acute cell lysis. Hyperphosphatemia, hyperkalemia, and hyperuricemia are indicative of tumor lysis syndrome. Calcium levels are decreased due to binding with free phosphorus and a depletion of calcium in the bloodstream. Sodium electrolyte levels are not as likely to be affected.

A 51-year-old female presents with concerns about a change in her cognition. She says she has difficulty retrieving words, loses her train of thought, and goes into a room and forgets why she came there. She also has had more frequent hot flashes and sleep disturbances. She still menstruates but has noticed a change from her previous pattern. A physical examination is unremarkable, and recent laboratory tests were all normal, including vitamin B12 and thyroid studies. Cognitive testing is normal. Which one of the following would be the most appropriate next step? A) Reassurance only B) CT of the head C) MRI of the brain D) Hormone therapy E) Referral to a neurologist

A Women experience subjective cognitive difficulties during their menopausal transition. This may include retrieving numbers or words, losing one's train of thought, forgetting appointments, and forgetting the purpose of behavior such as entering a room.

A 6-year-old female who recently moved to the United States from India requires a physical examination prior to entering the public school system. Her immunizations are up to date, including bacillus Calmette-Guérin vaccine at birth. Her family history is positive for her paternal grandfather being treated for latent tuberculosis infection. Her past medical history and a physical examination are otherwise unremarkable. Which one of the following would be most appropriate at this time? A) An interferon-gamma release assay (IGRA, QuantiFERON-TB Gold) B) A nucleic acid amplification test C) A tuberculin skin test, and follow-up in 72 hours D) Three acid-fast bacilli sputum samples and N95 mask use E) A chest radiograph

A recommend testing for tuberculosis with an interferon-gamma release assay (IGRA) rather than a tuberculin skin test (TST) in individuals >5 years of age who have received the bacillus Calmette-Guérin (BCG) vaccine. IGRA also ideal when the likelihood of the patient returning for a follow-up is low.

A 70-year-old female comes to your office for a routine health maintenance examination. Her past medical history is notable for well controlled hypertension and right knee osteoarthritis, and she underwent total knee replacement 1 year ago with an uncomplicated postoperative course. She mentions that she has an appointment to have a tooth filled and asks for your advice on antibiotics to prevent infection in her prosthetic joint. She has no medication allergies, has no previous history of joint infection, and is not immunocompromised. Which one of the following would be the most appropriate recommendation regarding antibiotic prophylaxis for her upcoming dental procedure? A) No antibiotic treatment B) A single dose of oral amoxicillin C) A 3-day course of oral amoxicillin D) A single dose of oral clindamycin (Cleocin) E ) A single dose of intravenous cefazolin

A t. Recent guidelines from the American Dental Association and the American Academy of Orthopaedic Surgeons recommend against the routine use of prophylactic antibiotics for dental procedures in patients with a history of joint replacement, except for situations in which infectious risk is increased, such as immunocompromise or a history of a previous joint infection.

A 67-year-old male sees you for a Medicare annual wellness visit. He tells you that his best friend had a stroke and he asks about his risk for stroke. He has no history of stroke, TIA, or neurologic symptoms. He has a family history of cardiovascular disease in his father, who had a myocardial infarction at age 65 and died from a thrombotic stroke at age 71. The patient exercises regularly and has a BMI of 27 kg/m2 . His only current medical condition is hyperlipidemia, and his cholesterol level is at goal on rosuvastatin (Crestor), 10 mg daily. He also takes aspirin, 81 mg daily. His blood pressure is 125/78 mm Hg. Based on U.S. Preventive Services Task Force guidelines, which one of the following would be most appropriate at this time? A) No additional testing for stroke risk B) Auscultation for carotid bruits C) Carotid duplex ultrasonography D) Magnetic resonance angiography E) CT angiography of the carotid arteries

A) No additional testing for stroke risk

A 28-year-old female presents for evaluation of nasal congestion, sneezing, watery eyes, and postnasal drip. This has been an intermittent issue for her every spring and she would like to manage it more effectively. Which one of the following treatments has been shown to be the most effective and best tolerated first-line therapy for this patient's condition? A) A leukotriene receptor antagonist B) Intranasal corticosteroid monotherapy C) Intranasal corticosteroids plus an oral antihistamine D) Inhaled corticosteroids E) Annual triamcinolone injections

B

Chronic kidney disease is defined by abnormal kidney structure or function lasting a minimum of A) 2 months B) 3 months C) 6 months D) 12 months E) 24 months

B

A 4-year-old male is brought to your office by his mother because of a 2-day history of watery diarrhea and vomiting and you diagnose acute gastroenteritis. On examination his mucous membranes are sticky and he has decreased tear production, but his overall appearance is normal and his eyes are not sunken. Using the Clinical Dehydration Scale, you estimate that he has mild (3%-6%) dehydration. Which one of the following should you recommend? A) Water as tolerated B) Half-strength apple juice followed by preferred fluids C) The bananas, rice, applesauce, and toast (BRAT) diet D) Intravenous fluids E) Metoclopramide (Reglan)

B A randomized, controlled trial has shown that initial rehydration with diluted apple juice followed by preferred fluids resulted in fewer treatment failures than use of a formal electrolyte solution. This is likely due to the increased likelihood that children will drink preferred fluids due to better taste, tolerability, and ease of administration

A 34-year-old male sees you because he was recently informed that a partner he had unprotected sex with last month has been diagnosed with HIV. You would advise this patient to initiate ongoing antiretroviral therapy A) immediately, because HIV testing is not necessary prior to initiation B) at the time of diagnosis of HIV infection C) when his CD4 cell count drops to <200 cells/ L D) when his CD4 cell count drops to <500 cells/ L E) when he develops an AIDS-defining illness

B ART should be prescribed at the time of diagnosis of HIV infection unless the patient has expressed a desire to not initiate treatment.

The mother of a 6-month-old infant is concerned that her child's feet are "deformed." On examination the heel bisector line is between the third and fourth digits on the right foot and on the third digit on the left foot. You attempt to flex the feet, and both appear to be rigid. Which one of the following would you recommend as a corrective intervention? A) Night splints B) Adjustable orthotic shoes C) Braces D) Physical therapy E) Surgical correction

B Adjustable orthotic shoes in infants who are not yet walking can be effective for the treatment of metatarsus adductus

A patient's office spirometry results demonstrate an obstructive pattern. This would be seen with which one of the following? A) Asbestosis exposure B) Cystic fibrosis C) Idiopathic pulmonary fibrosis D) Nitrofurantoin exposure E) Sarcoidosis

B All of the others are restrictive. Other examples of obstructive patterns include COPD, asthma, A1AT, and bronchiectasis.

A 25-year-old primigravida presents to your office in her second trimester with a 24-hour history of fever, cough, and myalgias. A nasal swab is positive for influenza A. She has a temperature of 38.6°C (101.5°F), a heart rate of 100 beats/min, a respiratory rate of 15/min, a blood pressure of 100/64 mm Hg, and an oxygen saturation of 98% on room air. On examination the patient is warm to the touch with mild cervical lymphadenopathy and moist mucous membranes. Her lungs are clear to auscultation bilaterally without wheezes, crackles, or rhonchi. A cardiovascular examination reveals a regular rate and rhythm without murmurs, rubs, or gallops. An abdominal examination is normal. Which one of the following would be the medication of choice for this patient? A) Baloxavir marboxil (Xofluza) B) Oseltamivir (Tamiflu) C) Peramivir (Rapivab) D) Zanamivir (Relenza)

B Antiviral medications are recommended for the treatment of influenza only within 48 hours of symptom onset (SOR A). However, in high-risk patient populations and in severe cases of disease, antivirals should be provided regardless of the duration of symptoms

A 1-month-old infant is brought to your office by her parents for routine follow-up. The infant was born at 35 weeks and 5 days gestation by normal vaginal delivery after induction of labor for maternal preeclampsia with severe features. The infant was discharged with the mother on the third day of life. There were no additional complications noted. Today the parents report exclusive breastfeeding and tell you the infant spits up after longer feedings. Voiding and stooling are as expected and the infant's development is normal. The weight at birth was at the 20th percentile, and today's weight is at the 25th percentile. Which one of the following would be the most appropriate intervention at this time? A) Caloric supplementation in addition to breastfeeding B) Iron supplementation C) Vitamin E supplementation D) Referral for early intervention

B Breastfed infants born before 37 weeks gestation should receive iron supplementation at 2 mg/kg/day after 1 month of life.

An otherwise healthy 21-year-old male sees you for follow-up after a hospitalization for pneumonia. This was his second pneumonia infection of the year. He reports a history of multiple sinus infections and upper respiratory infections over the years that were treated with antibiotics on an outpatient basis. Laboratory studies reveal a normal CBC and a decreased IgA level. A trial of pneumococcal polysaccharide vaccine (PPSV23, Pneumovax 23) reveals no measurable response. This presentation is most consistent with A) selective IgA deficiency B) common variable immunodeficiency C) severe combined immunodeficiency D) DiGeorge syndrome E) Wiskott-Aldrich syndrome

B CVID is the only immunodeficiency condition listed that can present later in life. SCID, DiGeorge, and Wiskott-Aldrich syndrome typically present prior to 6 months of age. CVID is a condition of impaired humoral immunity and thus should be considered in a patient this age in the setting of recurrent bacterial infections such as sinusitis or pneumonia. Blunted response to vaccination suggests impaired IgG.

A 44-year-old male with diabetes mellitus, hypertension, obesity, and chronic pain is on chronic opiate therapy. He comes to your office because of a lack of sex drive, decreasing strength, low overall energy levels, and hot flashes. After ruling out other causes you confirm that he has a low total testosterone level on two separate early morning laboratory tests. He would like to start testosterone therapy. Which one of the following would be the most appropriate next step? A) Order a PSA level and perform a digital rectal examination B) Order LH and FSH levels C) Order chromosomal studies D) Discuss risks and benefits of testosterone replacement therapy and start low-dose replacement E) Inform him that testosterone replacement therapy would not be beneficial for him because of its high risk

B Checking LH and FSH levels is recommended to evaluate for primary hypogonadism. If primary hypogonadism is present, chromosomal studies should be considered. Before initiating testosterone therapy, checking the patient's PSA level and performing a digital rectal examination are recommended, but in this case the initial workup is not yet complete.

A 6-year-old female is brought to your office by her parents for a routine well child visit. On examination the patient's height is 121 cm (48 in), which is the 91st percentile for her age, and her weight is 27 kg (59 lb), which is the 92nd percentile for her age. Her parents ask you about the recommended car safety restraints. Which one of the following would be an appropriate recommendation regarding vehicular safety for this child? A) She should use a rear-facing car seat B) She may use either a booster seat or a car seat C) She can safely wear a seatbelt without a booster seat when she reaches 127 cm (50 in) D) She should not use a car seat because her height exceeds the limit of common car seats E) She may sit in the front seat once she turns 11 years of age

B Children 4-8 years of age may be appropriately restrained in a car seat or booster seat, with a booster recommended when the child outgrows the forward-facing limit of the convertible or combination car seat. This child may still fit many car seats, though she is too large for rear-facing seats. Children can generally be safely restrained without a booster seat when their height reaches around 145 cm (57 in), though this lower limit can vary based on the specific vehicle. All children who ride in motorized vehicles should be restrained in the back seat until at least age 13

Which one of the following best explains the pathophysiology of cytokine storm? A) Anaphylaxis B) Immune dysregulation C) Immunodeficiency D) Normal physiologic response E) Serum sickness

B Cytokine storm or cytokine release syndrome is caused by the release of cytokines and is characterized by fever, tachypnea, headache, tachycardia, hypotension, rash, and/or hypoxia. Cytokine storm can be triggered by certain therapies, pathogens, cancers, autoimmune conditions, and monogenic disorders. The normal inflammatory response involves recognition of a pathogen or injury, activation of a proportional response, and a return to homeostasis. However, cytokine storm involves immune dysregulation and immune-cell hyperactivation in which an overabundance of cytokines can cause collateral damage that may be worse than the benefit from the immune response itself. I

A 60-year-old male is referred to you by his dermatologist for additional workup of a symmetric vesicular rash that was diagnosed as dermatitis herpetiformis. This rash is associated with A) food allergy B) gluten sensitivity C) HIV infection D) immunodeficiency E) internal malignancy

B Dermatitis herpetiformis is an immunologic response to ingested gluten and is pathognomonic for celiac disease. It may be the only presenting symptom of celiac disease, so there should be a high index of suspicion in patients presenting with a rash consistent with dermatitis herpetiformis. The papulovesicular rash is extremely pruritic and found on extensor surfaces such as elbows and knees, as well as the buttocks and scalp.

A 60-year-old male with diabetes mellitus and hypertension sees you for routine follow-up. He has no acute health concerns during today's visit. His current medications include metformin (Glucophage), lisinopril (Prinivil, Zestril), and hydrochlorothiazide. He smokes cigarettes and has a 40-pack-year smoking history. His vital signs and a physical examination are normal. An in-office dipstick urinalysis reveals 1+ blood and trace protein but is otherwise normal. Which one of the following would be the most appropriate follow-up? A) Repeat dipstick urinalysis in 3 months B) Microscopic urinalysis C) Renal ultrasonography D) CT urography E) Referral for cystoscopy

B Dipstick analysis alone is insufficient to diagnose microscopic hematuria, as blood that is seen on dipstick analysis may represent a false-positive result caused by myoglobinuria, hemoglobinuria, dehydration, exercise, menstrual blood, or povidone-iodine, as opposed to true hematuria. Thus, when the presence of blood is suggested by dipstick urinalysis, confirmation with microscopic analysis should be obtained.

A 30-year-old female presents with episodes of severe vertigo lasting 4-5 hours and associated with tinnitus, nausea, and vomiting. On examination the Dix-Hallpike maneuver is negative. She has no focal weakness, numbness, or paresthesia. She does not take any medications and her medical history is remarkable only for frequent headaches and seasonal allergies. Audiometry is abnormal due to low- and high-frequency hearing loss in her right ear, with preservation of the midrange. Which one of the following is the most likely diagnosis? A) Benign paroxysmal positional vertigo B) Meniere's disease C) Multiple sclerosis D) Vestibular migraines E) Vestibular schwannoma

B Episodic vertigo, hearing loss, and tinnitus are the classic triad of Meniere's disease. While nausea, vomiting, and headaches may coexist with the classic triad, these symptoms are not used to diagnosis the disease.

A 33-year-old gravida 2 para 1 presents to the hospital at 35 weeks estimated gestation with premature rupture of membranes. A decision is made to manage the pregnancy expectantly and delay delivery unless signs of infection or fetal distress are noted. Based on current evidence, expectant management rather than immediate delivery increases the risk of which one of the following complications? A) Cesarean delivery B) Antepartum or postpartum maternal hemorrhage C) Time spent in the neonatal intensive-care unit D) Neonatal sepsis E) Perinatal or infant mortality

B Expectant management decreases the risk of cesarean delivery, neonatal respiratory distress, mechanical ventilation, time spent in the neonatal intensive-care unit, and time spent in the hospital. Expectant management did increase the risk of maternal antepartum or postpartum hemorrhage and intrapartum fever

A 15-year-old male presents to the emergency department after falling on his bicycle and hitting his head while not wearing a helmet. He was dropped off by a friend's father because he was unable to reach his parents. The patient tells you that he has a headache and slight nausea but otherwise feels fine. The triage nurse is also unable to reach a parent or guardian. While continuing to try to reach the child's parent or guardian, which one of the following would be most appropriate regarding care for this patient? A) Observe him in the waiting room until his parent or guardian can be reached B) Complete the initial screening and provide any emergent treatment C) Ask two physicians to document an intent to treat prior to any screening or emergent treatment D) Contact a court official to obtain an emergency consent for medical treatment prior to any screening or emergent treatment E) Contact the child protective services agency and arrange for emergency guardianship prior to any screening or emergent treatment

B Federal law requires that the ED medical providers complete the initial medical screening and evaluation of minors even if parental consent cannot be obtained. In addition, treatment of emergencies is required, even without parental consent.

A 6-week-old female is brought to your office by her parents to establish care after the family recently moved from out of state. The infant was born at term after an uncomplicated normal spontaneous vaginal delivery but failed her initial newborn hearing screen in the right ear only. Both parents are confident that she is able to hear out of both ears because she turns her head toward their voices regardless of where they are standing. A physical examination is within normal limits. Which one of the following would be the most appropriate next step in response to this patient's abnormal hearing screen? A) No further testing B) A bilateral audiology evaluation before 3 months of age C) A bilateral audiology evaluation at 6 months of age D) A bilateral audiology evaluation at 12 months of age E) A bilateral audiology evaluation immediately before entering kindergarten

B For infants that fail the initial hearing screen in one or both ears, a repeat bilateral audiology evaluation should be performed before 3 months of age to ensure early identification of hearing loss and therefore maximize speech perception and development.

A 34-year-old female with asthma sees you for routine follow-up. She tells you that she uses her short-acting -agonist (SABA) approximately twice a week. Which one of the following management strategies would you recommend for prevention of exacerbations? A) Continued use of a SABA as needed B) An inhaled corticosteroid (ICS)/long-acting -agonist (LABA) as needed C) A daily maintenance ICS/LABA D) A daily maintenance ICS plus a SABA as needed E) A daily maintenance ICS plus a daily leukotriene receptor antagonist

B For patients with mild asthma, recent evidence has shown that an inhaled corticosteroid (ICS)/long-acting -agonist (LABA), such as budesonide/formoterol, as needed was as effective at preventing exacerbations as a daily maintenance ICS plus a short-acting -agonist (SABA) at one-fifth of the total corticosteroid dose. In addition, it was more effective at preventing exacerbations than continued use of a SABA alone as needed.

A 72-year-old female presents with pain, swelling, and decreased range of motion in her right great toe for several months. There is no history of injury or overuse. On examination the metatarsophalangeal joint is swollen and mildly tender, but not red. Dorsiflexion and plantar flexion are approximately 30°. A radiograph shows joint space narrowing and a small bone spur. Recommended management at this time would be A) stretching and strengthening exercises B) a rigid shoe insert C) ibuprofen D) a corticosteroid injection E) surgical referral

B Hallux rigidus affects as many as 50% of women and 40% of men by the age of 70. It is usually due to osteoarthritis of the metatarsophalangeal (MTP) joint and presents as decreased range of motion, swelling, and pain. With progression of the condition, flare-ups become more frequent and more severe, and it can be mistaken for gout. Initial treatment is restriction of motion across the MTP joint. A stiffening shoe insert does relieve pain and most patients see improvement without surgery. Custom orthotics, rigid inserts, or hard-soled shoes are options that are more effective than NSAIDs

Which one of the following is an indication for a radionuclide thyroid uptake scan for a patient with a single thyroid nodule confirmed by ultrasonography? A) The presence of antithyroid antibodies B) A low TSH level C) An elevated TSH level D) A normal TSH level and a tender nodule E) The presence of the thyroid nodule regardless of other findings

B If the TSH level is low, then a radionuclide scan is indicated. If the scan indicates hyperfunctioning of the nodule, then fine-needle aspiration is not necessary and radioactive iodine ablation is generally the treatment of choice.

A 68-year-old female presents with a 2-month history of watery diarrhea. She has not had any blood or pus in her stools, and the stools are not oily. She has not had any history of fever, chills, or weight loss, and has not traveled recently. She smokes one pack of cigarettes per day. Her medications include ibuprofen, sertraline (Zoloft), and pantoprazole (Protonix). A CBC, metabolic panel, C-reactive protein level, IgA anti-tissue transglutaminase level, total IgA level, and stool guaiac test are all normal. Which one of the following tests would be most likely to yield a diagnosis? A) Clostridioides (Clostridium) difficile toxin B) Colonoscopy C) Fecal calprotectin D) A stool culture E) Stool examination for ova and parasites

B In patients with chronic nonbloody diarrhea, the differential diagnosis includes microscopic (lymphocytic or collagenous) colitis. The mucosa appears normal on colonoscopy but a biopsy will show lymphocytic infiltration of the epithelium. The etiology of this is unknown but there are several risk factors to consider, including older age, female sex, and smoking status.

A 42-year-old male presents with a 3-month history of epigastric pain, bloating, and occasional vomiting after eating. He has not had any weight loss, blood in the stools, or difficulty swallowing. He does not report any significant acid reflux symptoms. Which one of the following would be the best management strategy for this patient? A) Implement lifestyle changes and follow up in 3 months B) Test for Helicobacter pylori and treat if positive C) Initiate a 2-month trial of proton pump inhibitor therapy D) Order esophagogastroduodenoscopy E) Order a barium swallow

B In patients younger than 55 years of age with no alarm symptoms, a test-and-treat strategy is effective and safe, with esophagogastroduodenoscopy reserved for patients not meeting these criteria (

You are providing end-of-life care for a 53-year-old female with end-stage colon cancer. Her family reports that she is having significant abdominal pain, nausea, and vomiting, and she is not able to tolerate oral intake. You suspect a malignant bowel obstruction. Which one of the following interventions would be most likely to significantly improve her symptoms? A) Medical cannabis B) Dexamethasone C) Morphine D) Octreotide (Sandostatin) E) Polyethylene glycol (MiraLAX)

B Malignant bowel obstruction is a common issue with gastrointestinal cancers. Corticosteroids can help alleviate these symptoms, which is the focus in end-of-life care. Corticosteroids have numerous beneficial effects in these situations, such as central antiemetic, anti-inflammatory, antisecretory, and analgesic effects.

Which one of the following is most appropriate for the initial management of volume overload due to an acute exacerbation of heart failure with preserved ejection fraction? A) Carvedilol (Coreg) B) Furosemide (Lasix) C) Lisinopril (Prinivil, Zestril) D) Sacubitril/valsartan (Entresto) E) Spironolactone (Aldactone)

B Management of HFpEF is treatment with diuretics, including loop diuretics.

A 63-year-old female sees you for follow-up after an emergency department (ED) visit. CT performed in the ED confirmed diverticulitis and she was treated as an outpatient. The discharge paperwork and radiology reports also indicate that she has a left superior adrenal nodule measuring 1.2 cm. Which one of the following concurrent conditions should prompt a hormonal workup? A) Hyperlipidemia B) Hypertension C) Renal cell carcinoma D) Rheumatoid arthritis E) Type 2 diabetes

B Most adrenal masses are benign nonfunctioning adenomas, but some are primary/functioning and secrete aldosterone and cortisol. A concurrent condition of renal cell carcinoma would suggest a metastatic lesion rather than a hyperfunctioning lesion. Hyperlipidemia, rheumatoid arthritis, and type 2 diabetes are not associated with excessive adrenal function.

A 62-year-old male is scheduled for CT of the chest with intravenous contrast in the next 48 hours. He has a long-standing history of degenerative joint disease in the right knee, coronary artery disease, and type 2 diabetes. His current medications are low-dose aspirin, metformin (Glucophage), and naproxen. In addition to discontinuing metformin prior to the procedure, which one of the following would you recommend for prevention of contrast-induced nephropathy? A) Discontinue aspirin B) Discontinue naproxen C) Start acetylcysteine D) Start mannitol (Osmitrol)

B NSAIDs such as naproxen should be withheld for 24-48 hours prior to a procedure involving venous or arterial administration of radiocontrast material. Avoidance of volume depletion and other nephrotoxic agents is also recommended. Administration of acetylcysteine or mannitol has not been shown to reduce the incidence of contrast-induced nephropathy.

A 70-year-old male presents with a 2-year history of gradually progressive exertional dyspnea associated with a dry cough and fatigue. A physical examination reveals bilateral basilar fine inspiratory crackles on lung auscultation and acrocyanosis. A chest radiograph demonstrates hazy opacities and reticular infiltrates of both lower lung fields. You suspect interstitial lung disease. Assuming that no underlying connective tissue disease is identified on serologic testing, which one of the following additional studies could confirm a diagnosis of idiopathic pulmonary fibrosis for this patient, potentially preventing the need for a subsequent lung biopsy? A) Spirometry B) High-resolution chest CT C) Polysomnography D) Echocardiography E) Right heart catheterization

B Of the options listed, only high-resolution chest CT has the potential to provide a specific diagnosis of IPF, which usually has a characteristic pattern of bilateral reticulation and honeycombing in the lung periphery and in the lower lobes termed usual interstitial pneumonia. Spirometry usually shows a restrictive pattern, although it may be normal in early disease or with comorbid emphysema. T

A 70-year-old female with a history of coronary artery disease, a femorofemoral bypass 3 years ago, and hypertension sees you for a follow-up visit. She has intermittent right arm pain that is worse with exercise. The pain increases with all arm exercises and improves with rest. The patient's blood pressure is 140/70 mm Hg in the left arm and 120/64 mm Hg in the right arm. Which one of the following would be the most appropriate next step? A) Radiographs of the right shoulder B) Arterial duplex ultrasonography of the upper extremities C) MR angiography D) No imaging, and referral to physical therapy

B PAD of the upper extremities is characterized by pain with exertion and can cause gangrene and ulceration. It is more common in patients who have had lower extremity occlusive disease. A blood pressure differential of 15 mm Hg between arms suggests stenosis and warrants further testing

A 4-month-old female is brought to your office by her parents as a new patient for a well child visit. The infant is healthy and the parents have no concerns. You have records from her previous physician that indicate the parents refused all vaccinations other than hepatitis B at birth. Which one of the following communication strategies is recommended to promote vaccine acceptance? A) Emphasize the benefits and downplay the risks of vaccines B) Clarify specific concerns and provide factual information C) Avoid future discussions with the family about vaccines D) Dismiss the family from the practice

B Parents and legal guardians of pediatric patients should be provided information on the benefits and risks of vaccination in clear and culturally sensitive language. Some parents express concerns about the need for, or safety of, certain vaccines.

A 35-year-old male with type 1 diabetes asks you what his hemoglobin A1c goal should be. His blood pressure and lipids are well controlled and he has not had any episodes of hypoglycemia. He recently saw an ophthalmologist and a podiatrist. You advise him that based on current American Diabetes Association recommendations his goal should be to keep his hemoglobin A1c below a threshold value of A) 6.0% B) 7.0% C) 8.0% D) 9.0%

B The American Diabetes Association recommends a hemoglobin A1c goal of <7.0% for nonpregnant adults. Glycemic targets may be higher in older adults, and in patients with functional impairments, limited life expectancy, or multiple comorbidities.

A 48-year-old runner presents with anterior knee pain. He says that the pain developed insidiously and is worse at the beginning of a run and immediately following a run. There is no history of injury. An examination suggests patellar tendinopathy. Which one of the following treatment modalities has the best evidence of long-term effectiveness in improving this condition? A) NSAIDs B) Eccentric quadriceps exercises C) Corticosteroid injection of the infrapatellar bursa D) Injection of the tendon with a sclerosing agent E) Surgical excision of the tendon

B Patellar tendinopathy may persist for years and may be refractory to treatment. Eccentric quadriceps-strengthening exercises have the best evidence for long-term improvement of the condition.

A 58-year-old male is brought to your office by his wife for follow-up after an urgent care clinic visit 3 days ago where he was diagnosed with pneumonia based on a chest radiograph. He has been taking his antibiotics as prescribed. He had shaking chills last night, and this morning at home he had a fever and has been very fatigued and slightly confused. He has a blood pressure of 110/70 mm Hg, a respiratory rate of 27/min, and a temperature of 38.7°C (101.7°F). Laboratory studies reveal elevated WBCs and elevated creatinine and lactate levels. Which one of the following would be the most appropriate next step? A) Adjust his antibiotic therapy to cover health care-associated pneumonia B) Obtain intravenous access and begin fluid resuscitation C) Order a procalcitonin level D) Order repeat chest radiography E) Order CT of the chest

B Patient has sepsis - needs to be fluid resuscitated.

Polypharmacy increases the risk of adverse health outcomes. According to the Choosing Wisely campaign, adding to a threshold of how many medications in a patient's regimen should prompt a thorough review to determine if any of the medications can be discontinued? A) 3 B) 5 C) 7 D) 10

B Polypharmacy = 5 or more meds

An asymptomatic 53-year-old female recently underwent a right breast lumpectomy and radiation therapy with curative intent. Over the next 5 years routine surveillance should include A) annual unilateral left breast mammography B) annual bilateral mammography C) annual bilateral mammography and radionuclide bone scans D) biannual bilateral mammography E) biannual bilateral mammography and annual radionuclide bone scans

B Primary care physicians should ensure that their patients who have undergone treatment for breast cancer follow the recommendations of their oncologist, as well as receive a history evaluation and health maintenance examination every 3-6 months for 3 years, every 6-12 months for 2 more years, and then on an annual basis. For ongoing surveillance only annual mammography is recommended (SOR A), which is bilateral in breast-conserving therapy and unilateral following a mastectomy.

A 66-year-old male presents to your office with a 1-week history of dyspnea with minimal exertion but no chest pain. He has had mild hemoptysis. An examination reveals a pulse rate of 100 beats/min but no other remarkable findings. A chest radiograph, CBC, and metabolic panel are normal, but his D-dimer level is elevated at 750 ng/mL (N <500). Which one of the following would be the next step in the evaluation? A) A BNP level B) CT pulmonary angiography C) An EKG D) Pulmonary arteriography E) A ventilation-perfusion scan

B Pulmonary embolus is reliably diagnosed with CT pulmonary angiography (CTA), but there is now a simple diagnostic algorithm to reduce the reliance on CTA. The simplified recommendations for ordering CTA are a D-dimer 1000 ng/mL, or a D-dimer that is >500 ng/mL and hemoptysis, signs of deep vein thrombosis, or a suspicion that pulmonary embolism is the most likely diagnosis.

A 62-year-old male was hospitalized for an upper gastrointestinal (GI) bleeding episode and the gastroenterologist arranges for you to provide follow-up care. The patient's medications include atorvastatin (Lipitor), metformin (Glucophage), lisinopril/hydrochlorothiazide (Zestoretic), citalopram (Celexa), and omeprazole (Prilosec). Which one of the following medications in his current regimen increases the risk of an upper GI bleeding episode? A) Atorvastatin B) Citalopram C) Lisinopril/hydrochlorothiazide D) Metformin E) Omeprazole

B SSRIs such as citalopram increase the risk of upper gastrointestinal (GI) bleeding by 55%, according to meta-analysis studies including more than 1 million subjects (

An otherwise healthy 72-year-old male presents with a 4-week history of catching and triggering of his right middle finger. When he awakens in the morning the finger is locked in flexion at the proximal interphalangeal joint and he has to manually extend the finger. He enjoys playing golf and painting, both of which are compromised by the triggering of his finger. He has not had any pain or numbness. Which one of the following would you tell him regarding his treatment options for this condition? A) Less than 15% of cases spontaneously resolve by 1 year B) Splinting of the affected finger can result in complete resolution C) NSAID injections have been found to be more effective than corticosteroid injections D) Surgical release is the most cost-effective treatment

B Splinting, which is a first-line treatment, has been shown to be effective. Single-joint orthoses at either the metacarpophalangeal or the proximal interphalangeal joint can be effective. Duration of splinting can range from 6 weeks to 3 months.

A 60-year-old female sees you for evaluation of a recent syncopal episode. She tells you that she recently fainted during a routine walk with a friend, who told her that everything seemed fine and suddenly the patient fell to the ground and was unresponsive. The patient does not recall having any symptoms before the episode. She did not have any associated symptoms, abnormal movement, or incontinence and was not injured in the fall. The episode lasted a few seconds and then she regained consciousness, stood, and finished the walk without difficulty. She has not had any similar episodes before this. Her past medical history and family history are unremarkable. She has been in good health and takes no medications except a multivitamin each day. An examination is normal and an EKG shows normal sinus rhythm with no other abnormalities. Which one of the following would most likely help confirm the diagnosis? A) Orthostatic blood pressure measurements B) An external cardiac event monitor C) Ultrasonography of the carotid arteries D) CT of the head E) MRI of the brain

B Syncope is generally classified into three broad categories: cardiac, neurally mediated, and orthostatic hypotension. The history of an abrupt unprovoked episode of syncope in this patient suggests a cardiac source. The most frequent cause of cardiac-associated syncope is an arrhythmia. The fact that this patient's syncope occurred during exercise (walking) also suggests an arrhythmia

A 23-year-old primigravida comes to your office for her initial obstetric visit. She is at 13 weeks gestation based on the dates of her last menstrual period. She is a nonsmoker and does not drink alcohol or use illicit substances. Her vital signs are remarkable for a blood pressure of 142/92 mm Hg and a BMI of 32 kg/m2 . She says that she has been diagnosed with hypertension in the past but has not taken any medications for it. In addition to a prenatal vitamin, which one of the following would you recommend for her? A) No additional medications B) Aspirin C) Ferrous sulfate D) Folic acid E) Labetalol (Trandate)

B The U.S. Preventive Services Task Force (USPSTF) recommends prescribing low-dose aspirin after 12 weeks gestation for asymptomatic women at high risk for preeclampsia. Women at high risk include those with a history of preeclampsia, chronic hypertension, multiple pregnancy, type 1 or 2 diabetes, renal disease, autoimmune disease, or any combination of these.

The most common symptom of obstructive sleep apnea is A) cough B) excessive sleepiness C) leg swelling D) palpitations E) weight gain

B The most common presenting symptom is excessive sleepiness; patients may also present with fatigue and lack of energy. Cough, leg swelling, palpitations, and weight gain are not among the most common presenting symptoms of OSA.

A healthy 36-year-old female who is a nonsmoker sees you for a routine well woman examination. She has been sexually active with five partners in the last 2 years. She has never had an abnormal Papanicolaou (Pap) smear. Last year's Pap test with high-risk HPV co-testing was negative. You review her immunization status and note that she received the influenza vaccine last fall. Which one of the following vaccines that this patient has never previously received would you recommend for her? A) Hepatitis A vaccine B) HPV vaccine (Gardasil 9) C) Meningococcal polysaccharide conjugate vaccine (Menactra) D) Pneumococcal polysaccharide vaccine (PPSV23, Pneumovax 23) E) Recombinant zoster vaccine (Shingrix)

B The only vaccine indicated for this patient would be the HPV vaccine, which the CDC recommends as a routine vaccination for all patients starting at 11 or 12 years of age through 26 years of age but can also be considered in adults 27-45 years of age who have not previously received the vaccine and are most likely to benefit

A 30-year-old female who is an established patient calls your office to request a test for COVID-19. The patient spent several hours inside the home of another individual who just received a positive COVID-19 test result. She states that her sense of taste seems diminished, but she has no respiratory symptoms and otherwise feels well. Which one of the following is the typical incubation period for COVID-19? A) 1 day B) 5 days C) 14 days D) 30 days

B The typical incubation period for COVID-19 is approximately 4-5 days, though it can range from 1-14 days

The U.S. Preventive Services Task Force recommends one-time screening for abdominal aortic aneurysm in A) men 55-75 years of age who have ever smoked B) men 65-75 years of age who have ever smoked C) men and women 65-75 years of age who have ever smoked D) men and women 65-75 years of age who currently smoke

B There is a moderate net benefit for screening for AAA in men 65- to 75-years old who have ever smoked (defined as >100 lifetime cigarettes)

Which one of the following vitamins is a well established therapy for the treatment of nausea and vomiting in pregnancy? A) Vitamin A B) Vitamin B6 C) Vitamin B12 D) Vitamin C E) Vitamin E

B These options include vitamin B6 (pyridoxine), over-the-counter antihistamines such as doxylamine, and natural ginger (<1500 mg daily). In addition, combination doxylamine/pyridoxine is approved by the FDA for the prevention of nausea and vomiting in pregnancy. Prescription antiemetics such as metoclopramide or trimethobenzamide are reserved for severe or refractory cases.

You receive a call from the home health nurse who is caring for a bedridden 57-year-old male with progressive multiple sclerosis. She is concerned that he has a weak cough reflex and may not be swallowing safely. She has not witnessed an aspiration event. He was hospitalized for pneumonia 4 months ago. While awaiting the results of a full swallow evaluation, which one of the following is the most appropriate intervention to prevent recurrent pneumonia in this patient? A) The use of chlorhexidine mouthwashes B) A mechanical soft diet with thickened liquids C) Swallowing exercises D) Antibiotic therapy for 24 hours E) Placement of a nasogastric tube

B This patient is at risk for aspiration pneumonia due to his neurologic disease and impaired cough reflex. A swallow evaluation is appropriate. A mechanical soft diet with thickened liquids is recommended rather than pureed foods and thin liquids.

A 5-year-old female is brought to your office by her mother because she has not wanted to eat or drink anything since she woke up this morning. She has also had a temperature of 100.2°F for the last 2 days. She usually attends day care 3 days a week but has been kept at home because of the fever. She is up to date on all age-appropriate recommended immunizations. On examination you notice several oral ulcerations that are painful to palpation, and several erythematous, vesicular lesions on the patient's palms. Which one of the following is the most likely diagnosis? A) Erythema multiforme B) Hand-foot-and-mouth disease C) Herpetic gingivostomatitis D) Oral candidiasis E) Varicella

B Hand-foot-and-mouth disease is a viral illness caused by human enteroviruses and coxsackie viruses that presents in the spring to the fall, generally in children <10 years of age. It is characterized by a low-grade fever, uncomfortable oral lesions, and a papular to vesicular rash on the hands and soles of the feet. Hydration and pain control with acetaminophen or ibuprofen are the mainstays of treatment.

A 30-year-old female with type 2 diabetes and obesity sees you for follow-up. She has experienced several episodes of symptomatic hypoglycemia, and because of this she stopped all of her medications except metformin (Glucophage). Her hemoglobin A1c has increased to 8.4%. Which one of the following would be the best additional treatment for this patient? A) Basal insulin (Lantus) B) Rapid-acting insulin (Humalog) C) Exenatide (Byetta) D) Glipizide (Glucotrol) E) Repaglinide

C Exenatide is a GLP-1 receptor agonist that is not associated with hypoglycemia and can also assist with weight loss, which would be helpful in this patient with obesity. All of the other listed medications, including both types of insulin, sulfonylureas, and meglitinides, can be associated with hypoglycemia.

A 72-year-old male with a history of hypertension, heart failure, and chronic kidney disease sees you for evaluation of gradually worsening lumbar pain. The pain worsens with walking but improves when he sits. He says that the pain radiates to the buttocks and down the right leg, especially with activity. He has not had any fevers, chills, or new urinary symptoms. MRI indicates severe degenerative changes resulting in moderate to severe canal stenosis at the L4-L5 level. Which one of the following would be most appropriate at this point? A) Oral diclofenac, 75 mg twice daily B) Oral pregabalin (Lyrica), 75 mg twice daily C) Physical therapy D) Referral to an orthopedic surgeon for elective surgical resolution E) Referral to a neurosurgeon for urgent surgical resolution

C Focused physical therapy has the best evidence for initial management.

During a newborn examination the patient's mother asks what she can do to decrease the risk of food allergies in her newborn son. She tells you that there is no family history of atopic dermatitis or asthma but she has a cousin with a peanut allergy. The remainder of the examination is unremarkable. You tell her that food allergy risk can be reduced by A) breastfeeding for at least 1 year B) using soy-based formula instead of cow's milk-based formula C) introducing peanut-containing foods when solids are started D) avoiding all house pets E) avoiding a day care setting

C

A 3-year-old male undergoes surgery for intestinal malrotation (volvulus). A preoperative comprehensive metabolic panel was normal. The patient's postoperative potassium level is 4.6 mEq/L (N 3.4-4.7). In addition to maintenance potassium, which one of the following fluids should be administered in the postoperative period? A) 0.2% NaCl with 5% dextrose B) 0.45% NaCl with 5% dextrose C) 0.9% NaCl with 5% dextrose D) 3% saline E) 5% dextrose in water

C AAP recommends the use of isotonic solutions with adequate potassium chloride and dextrose for MIVF in children.

You diagnose nonvalvular atrial fibrillation in a 54-year-old male. His CHA2DS2-VASc score is 2. Which one of the following should you recommend as first-line therapy for stroke prevention? A) No antithrombotic therapy B) Aspirin C) A direct oral anticoagulant D) Low molecular weight heparin E) A vitamin K antagonist

C DOACs are first line for prevention of stroke in nonvalvular a fib with CHADS-VASc score ≥2 in men,, ≥3 in women

A 64-year-old female presents to the emergency department with a 10-day history of increasing shortness of breath and mild tachycardia. On examination she has an oxygen saturation of 75% on room air. Which one of the following additional findings would suggest a diagnosis of acute respiratory distress syndrome (ARDS)? A) Improved oxygen saturation with supplemental oxygen B) Improvement of her symptoms with diuretic therapy C) Bilateral airspace opacities seen on a chest radiograph D) A flattened diaphragm seen on a chest radiograph E) A right lower lobe infiltrate seen on a chest radiograph

C Acute respiratory distress syndrome (ARDS) will often present similarly to pneumonia and heart failure with dyspnea, hypoxemia, and tachypnea. ARDS typically does not respond to supplemental oxygen or diuretic therapy. Patients decompensate quickly and usually require mechanical ventilation. Chest radiographic findings include bilateral airspace opacities but not a localized infiltrate as with pneumonia, venous congestion or cardiac enlargement as with heart failure, or a flattened diaphragm

A 15-year-old female is brought to your office by her parents for evaluation because they are concerned about her restrictive eating patterns and weight loss. The patient is unconcerned about these issues and says that she feels well and does not need any evaluation. Her parents tell you that for the past 6 months she has had an increasingly restricted diet to the point that she now drinks only water and eats only vegetables and roasted chicken or turkey. They report that she looks much thinner now than she did 6 months ago, but they are uncertain how much weight she has lost. She says that she does not feel depressed or anxious and she is doing well in school. On examination she has a height of 163 cm (64 in) and a weight of 43 kg (95 lb), with a BMI of 16 kg/m2 . She has a pulse rate of 52 beats/min and a blood pressure of 102/68 mm Hg while seated and 84/58 mm Hg while standing. Evaluation of her teeth shows significant erosion of the enamel. When considering the psychotherapy aspect of care for this patient, which one of the following is preferred for treatment of her condition? A) Cognitive-behavioral therapy B) Dialectical behavioral therapy C) Family therapy D) Interpersonal therapy E) Psychodynamic therapy

C Anorexia nervosa. Psychotherapy is the foundation of treatment and parental involvement is key for children and adolescents.

A 47-year-old female presents with a 2-month history of generalized left shoulder pain. She does not have any ongoing illnesses. On examination you note generally impaired range of motion in the shoulder, but a radiograph of the shoulder is normal. You diagnose adhesive capsulitis. Which one of the following tests would be most appropriate at this time? A) Antinuclear antibody B) Erythrocyte sedimentation rate C) Hemoglobin A1c D) IgA tissue transglutaminase E) MRI without contrast

C Both diabetes mellitus and hypothyroidism have been found to have a prevalence of 25%-50% in patients with adhesive capsulitis, which is also known as frozen shoulder. Consideration should be given to testing for both of these conditions when making the diagnosis of adhesive capsulitis

An 8-year-old female is brought to your office because of left arm pain after she fell down on the sidewalk while roller skating. She has pain, swelling, and a mild deformity of her distal forearm over the radius. Posteroanterior and lateral radiographs confirm an incomplete compression fracture of the distal radius. In addition to a short arm splint, which one of the following would be appropriate management of this fracture? A) Ultrasonography in 3 weeks B) Repeat radiography in 4 weeks C) Return to activity in 4 weeks if she is pain free D) Follow-up and reevaluation in 6 weeks E) Referral to an orthopedist

C Buckle fracture = compression fracture of the distal radius. These are inherently stable fractures. Tx is short arm immobilization. No need for repeat imaging if there is no longer any pain or tenderness with palpation after 4 weeks of splinting.

A 23-year-old male with opioid use disorder requests buprenorphine therapy. He is still actively using immediate-release oxycodone (Roxicodone) and he took a dose 2 hours ago. This patient should begin buprenorphine induction A) now B) in 2 hours C) 8-12 hours after his last opioid use D) 24 hours after his last opioid use E) 1 week after his last opioid use

C Buprenorphine is a partial opioid agonist. In order to reduce the risk of precipitated withdrawal, buprenorphine induction should begin once the patient is exhibiting signs of mild to moderate withdrawal, usually 8-12 hours after the last opioid use

In a patient with persistent respiratory symptoms, which one of the following pulmonary function abnormalities after bronchodilator administration is required for the diagnosis of COPD? A) Low residual volume B) Low total lung capacity C) An FEV1/FVC ratio <0.70 D) An FEV1 <85% of predicted E) A peak flow <90% of predicted

C COPD is defined by FEV1/FVC <0.7

A 32-year-old male comes to your office for a follow-up visit after spontaneously passing a renal stone. This is his third such episode. Analysis of a previous stone showed that it was composed of calcium oxalate. After the first episode he increased his fluid intake and is drinking more than 3 L of water daily. He does not take any medications. An examination is unremarkable. Which one of the following should you recommend at this time? A) A very-low-calcium diet B) Furosemide (Lasix) C) Potassium citrate D) Probenecid

C Calcium stones, composed of either calcium oxalate and/or phosphate, account for up to 90% of all stones in adults in developed countries. Increasing fluid intake to 2.5-3 L/day is the most important lifestyle modification to prevent recurrent kidney stones. A diet rich in fiber and vegetables with normal calcium content (1-1.2 g/day), limited sodium intake (4-5 g/day), and limited animal protein intake (0.8-1 g/kg/day) is strongly encouraged. Reduction of BMI by dietary modification and increased exercise is also recommended. Citrate supplementation with potassium citrate is recommended for preventing calcium stones that recur despite lifestyle modification

A 74-year-old male with type 2 diabetes, severe peripheral artery disease, and a history of tobacco use is admitted to the hospital with wet gangrene of his right foot that does not improve with appropriate wound care and antibiotics. His vascular surgeon recommends a below-the-knee amputation of the right leg but the patient has not given his consent. Which one of the following would suggest that this patient lacks capacity to make this decision? A) He has a fear of having surgery B) He wants to consult with his family first C) He consistently believes that his foot will improve D) He understands that he may die if he forgoes the amputation E) He states that his brother had the same surgery and died a week later

C Capacity to make a medical decision requires that a patient understands the risks, benefits, and alternatives to a specific treatment recommendation.

A 71-year-old female with a history of well controlled hypertension, diabetes mellitus, and osteoporosis presents with a 2-day history of fever, chills, and a productive cough. She lives at home with her husband, who has not noted any confusion but says she has been weak and unable to bathe herself. On examination the patient has a temperature of 38.2°C (100.8°F), a blood pressure of 110/68 mm Hg, unlabored respirations at a rate of 22/min, and an oxygen saturation of 94% on room air. You note that she has good air entry, there are no abnormal breath sounds, and there is no egophony or increased fremitus. The cardiovascular examination is unremarkable. Labs: WBC 14K Hgb 12.5 Plt 250K Cr 1.0 BUN 14 PA/Lateral CXR show infiltrate in RML. Which of the following would be the most appropriate treatment in this patient? A) Azithromycin (Zithromax) B) Amoxicillin plus metronidazole (Flagyl) C) Amoxicillin/clavulanate (Augmentin) plus azithromycin D) Azithromycin plus levofloxacin E) Clindamycin (Cleocin) plus doxycycline

C For outpatients with comorbidities, amoxicillin/clavulanate is a possible treatment option, but it should be paired with a macrolide. Macrolides such as azithromycin are the treatment of choice for previously healthy outpatients with no history of antibiotic use within the past 3 months.

A 3-year-old male is brought to your office by his mother because he stepped on a large wooden splinter that broke off at the surface of his left foot and since then he has been avoiding walking on that foot. On examination the bottom of the left heel is red and inflamed. Which one of the following would be most appropriate initially to visualize the splinter? A) Radiography B) Fluoroscopy C) Ultrasonography D) CT E) MRI

C Foreign bodies can be challenging to both detect and remove, especially in younger children. Ultrasonography is good for detecting radiolucent material such as wood or vegetation. MRI can also be used but is more expensive and not as readily available, and may be dangerous if metal is present. There is no radiation exposure with either of these modalities. Plain radiography creates minimal exposure to radiation and can detect radiopaque materials such as glass and metal but cannot detect vegetative materials.

Which one of the following is the most common cause of koilonychia (spoon-shaped nails)? A) Chronic pulmonary disease B) Hyperthyroidism C) Iron deficiency anemia D) Onychomycosis E) Psoriasis

C Iron deficiency anemia is the most common cause of koilonychia, which is also known as spoon nail because it appears as a central depression in the nail that curves outward away from the nailbed, giving the nail the appearance of a spoon.

A 15-year-old basketball player presents with a 3-week history of bilateral knee pain that is greater in the left knee. The pain increases with jumping, walking down stairs, and kneeling. He has not had any recent injury. He tells you that he has grown more than 5 inches in the past year. A physical examination is notable for tight quadriceps and hamstrings, and tenderness to palpation over the tibial tuberosity bilaterally. Which one of the following is the most likely diagnosis? A) Anterior cruciate ligament tear B) Medial meniscus tear C) Osgood-Schlatter disease D) Patellar sleeve fracture E) Patellofemoral pain syndrome

C It most often occurs in children or adolescents who are rapidly growing. Rapid growth results in bone lengthening, which occurs more rapidly than lengthening of the associated muscle and tendons. Osgood-Schlatter disease is one type of apophysitis affecting the patellar tendon attachment at the proximal tibia.

A 7-year-old male is brought to the urgent care clinic with a 2-day history of fever and sore throat, with no associated cough. His temperature is 38.3°C (100.9°F) and a rapid antigen test confirms a group A -hemolytic Streptococcus infection. A prescription for penicillin is sent to the pharmacy, but the medication is never picked up due to a lack of transportation. The patient is brought to your office 2 weeks later with a fever, joint pain, shortness of breath, and chest pain. His vital signs are significant for a temperature of 38.8°C (101.8°F) and a heart rate of 118 beats/min. On examination the patient's affect is appropriate, he has a 3/6 holosystolic murmur heard best over the apex, and he has tenderness and swelling of his knees bilaterally and of his left ankle. An antistreptolysin O titer is positive, his erythrocyte sedimentation rate is 124 mm/hr (N <10), and a chest radiograph is significant for cardiomegaly. Which one of the following would be the most appropriate therapy? A) Hydroxychloroquine (Plaquenil) B) Methylprednisolone acetate (Depo-Medrol) C) Naproxen D) Intravenous immunoglobulins E) Plasmapheresis

C Meets Jones criteria for ARF. NSAIDS such as naproxen can provide significant relief and should be administered as soon as ARF is diagnosed. Tx with corticosteroids, IV immunoglobulins, and plasmapheresis is not considered appropriate for ARF but may be indicated for management of pediatric autoimmune neuropysch disorderes associated with streptococcal infections (PANDAS)

Which one of the following types of complementary/integrative therapy is a form of mental training that requires calming of thoughts with the goal of achieving a state of detached observation? A) Aerobic exercise B) Cognitive-behavioral therapy C) Mindfulness-based meditation D) Tai chi and qi gong E) Yoga

C Mindfulness-based meditation is a form of mental training that requires calming of thoughts with the goal of achieving a state of detached observation. R

An 18-year-old female comes to the urgent care clinic because of worsening nausea and vomiting, itching, and a dry cough that began about 30 minutes after she ate lunch at a nearby restaurant. She tells you that she did not experience any choking while eating her lunch, and she has not had any dysphagia, rash, or diarrhea. She takes no medications, and her past medical history is significant only for a severe nut allergy. She says that she was feeling well before today. An examination is notable only for a blood pressure of 88/60 mm Hg, mildly labored breathing, and bilateral expiratory wheezes. At this point you would administer A) albuterol (Proventil, Ventolin) B) diphenhydramine (Benadryl) C) epinephrine D) hydroxyzine (Vistaril) E) methylprednisolone (Medrol)

C Most anaphylactic reactions occur outside of the hospital setting, and early treatment decreases both hospitalizations and mortality. This patient presents with respiratory, dermatologic, cardiovascular, and gastrointestinal symptoms, which are common in anaphylaxis.

A 45-year-old male comes to your office for a routine health maintenance examination. He reports frequently drinking five 12-oz beers per day despite trying to cut down. He tells you that he had an abnormal liver profile in the past and sometimes drives under the influence of alcohol. He asks for your help to decrease his alcohol consumption. Which one of the following medications would be most likely to help this patient decrease his alcohol use? A) Baclofen intrathecal (Lioresal) B) Disulfiram (Antabuse) C) Naltrexone (Vivitrol) D) Pregabalin (Lyrica) E) Sertraline (Zoloft)

C Naltrexone has been shown to decrease heavy drinking, daily drinking, and the amount of alcohol consumed

While on rounds in the newborn nursery, you receive a call about a 2-day-old infant born at 39 weeks gestation. According to the American Academy of Pediatrics standard treatment guidelines for infants at high risk of bilirubin encephalopathy, the infant has an elevated total serum bilirubin level that is approaching the threshold for initiating phototherapy. Which one of the following additional factors would be the strongest indication for phototherapy in this infant? A) East Asian ethnicity B) Exclusive breastfeeding C) A positive direct antibody titer (Coombs test) D) A sibling with a history of neonatal jaundice E) Untreated maternal group B streptococcal colonization

C Of the options listed, hemolysis, which is associated with a positive direct antibody titer (Coombs) is the most significant risk factor for developing acute bilirubin encephalopathy and therefore impacts the treatment threshold for initiations of phototherapy.

A 52-year-old male with known hypertension and hyperlipidemia comes to your office for a follow-up visit. His last visit was more than a year ago. He was unemployed for several months and lost his health insurance. Two months ago he ran out of his medications, which included amlodipine (Norvasc), hydrochlorothiazide, and atorvastatin (Lipitor). He says that he feels fine and has not had any chest pain, changes in vision, difficulty breathing, or lower extremity edema. He is a nonsmoker, and he does not drink alcohol or use illicit drugs. He drinks one cup of coffee daily. He does not take any over-the-counter medications. On examination his vital signs include a blood pressure of 190/120 mm Hg, a pulse rate of 80 beats/min, and an oxygen saturation of 96% on room air. You recheck his blood pressure after he sits quietly for 30 minutes and there is no significant change. A physical examination, including fundoscopy, is normal. Which one of the following would be the most appropriate next step? A) Administer clonidine (Catapres), 0.1 mg orally, and recheck his blood pressure in 30 minutes B) Administer nifedipine (Procardia), 60 mg orally, and recheck his blood pressure in 30 minutes C ) Order laboratory studies to look for end-organ damage, and tell him to restart his previous medications D) Admit him to the intensive-care unit for intravenous treatment to lower his blood pressure E) Call 911 and have him transported to the emergency department

C Patients who are asymptomatic with persistently elevated blood pressures can be safely treated with oral antihypertensives with close follow-up (

A 42-year-old transgender male comes to your office for a routine health maintenance examination. The patient's current medications include testosterone enanthate (Xyosted), 100 mg subcutaneously every 7 days, for gender affirmation, and medroxyprogesterone acetate (Depo-subQ Provera 104), 104 mg subcutaneously every 12 weeks, for contraception. The patient has no current chronic health conditions and no current sexual partners, but he has previously been sexually active with men and women. Which one of the following health conditions is more likely to occur in this patient compared to a female cisgender patient? A) Anemia B) Cervical cancer C) Dyslipidemia D) Kidney disease E) Venous thromboembolism

C Risks of testosterone therapy include more atherogenic lipid profiles, an increase in blood pressure, and erythrocytosis (rather than anemia). Severe liver dysfunction is unusual at therapeutic dosages but is a concern at dosages above the recommended therapeutic range. Testosterone therapy has not been associated with cervical cancer, kidney disease, or venous thromboembolism (VTE).

You diagnose major depressive disorder in a 69-year-old male. He has benign prostatic hyperplasia and is treated with prazosin (Minipress) for nocturia but he is otherwise healthy. He agrees to cognitive-behavioral therapy and starting an antidepressant but is concerned about side effects, especially falls. Which one of the following would be the safest medication for this patient? A) Amitriptyline B) Duloxetine (Cymbalta) C) Escitalopram (Lexapro) D) Paroxetine (Paxil) E) Phenelzine (Nardil)

C SSRIs are safe in terms of side effects for older adults. Avoid paroxetine due to higher likelihood of adverse effects that include sedation and orthostatic hypotension.

A 6-month-old male is brought to your office by his mother for a well child examination. The mother does not have any concerns. Interactions between the mother and child are appropriate and the child appears well. Which one of the following screenings is recommended at this visit? A) Autism B) Iron deficiency C) Maternal depression D) Otoacoustic emissions (OAE) testing

C Screen for maternal depression at the 1-, 2-, 4-, and 6-month WCC. AAP recommends screening for iron deficiency at 12 months, but USPSTF finds insufficient evidence.

A 68-year-old male presents with a burn on his lower leg after trying to light a bonfire with kerosene. Examination of the affected leg reveals the presence of blistering, along with a denuded central area that does not blanch with pressure. The underlying fat and connective tissue are not involved. Which one of the following is the proper classification of this burn? A) Superficial burn B) Superficial partial-thickness burn C) Deep partial-thickness burn D) Full-thickness burn

C Superficial partial-thickness burns blister and blanch with pressure. Deep partial-thickness burns blister, but do not blanch with pressure. Full-thickness burns extend through the entire dermis and into the underlying tissues, and they are dry and leathery

An 82-year-old female with atrial fibrillation treated with digoxin is started on verapamil sustained-release capsules (Calan SR) for hypertension and angina. Although she initially tolerates the medication and has a good clinical response, when you see her 1 month later she has lost 3 kg (7 lb) and reports persistent anorexia and nausea over the past 2-3 weeks. A serum chemistry profile, TSH level, and CBC are normal. Her serum digoxin level is 1.4 ng/mL (therapeutic range 0.8-1.5). Her vital signs are stable and a physical examination is notable only for rate-controlled atrial fibrillation. Which one of the following would be most appropriate at this point? A) Prescribe a therapeutic trial of an H2-blocker B) Order an upper gastrointestinal contrast study C) Withhold digoxin for several days and reinstitute at a lower dosage if necessary D) Order imaging to look for a central nervous system abnormality

C Symptoms of digitalis toxicity, common in elderly patients even with therapeutic range of digoxin. Verapamil will increase the serum level of digoxin, which will reach a new steady state level in several days. Therefore, the side effects may not occur for several days while the level is increasing.

A 12-year-old male with type 1 diabetes is brought to your office for routine follow-up. Laboratory work performed prior to the appointment shows an LDL-cholesterol level of 120 mg/dL. In addition to counseling the patient on a heart-healthy diet and daily physical activity, which one of the following would you recommend? A) No additional measures B) Fish oil supplements C) Atorvastatin (Lipitor) D) Ezetimibe (Zetia) E) Gemfibrozil (Lopid)

C T1D has high risk for developing CVD. Current guidelines recommend initiating a statin, in addition to education regarding a healthy diet and physical activity, for pediatric patients in this high-risk category with an LDL-cholesterol level >100 mg/dL.

A 65-year-old female is diagnosed with osteoporosis based on a screening DXA scan. After a shared decision-making discussion you decide to initiate treatment with alendronate (Fosamax). Which one of the following would you recommend for the duration of pharmacologic treatment? A) 1 year B) 3 years C) 5 years D) 10 years E) Lifelong treatment

C The American College of Physicians evidence-based guideline recommends a maximum treatment duration of 5 years with alendronate.

A 60-year-old male comes to your office with a 1-year history of the gradual onset of mild fatigue and dyspnea. There are no symptom triggers. He has a 20-pack-year history of cigarette smoking but stopped at age 35. An examination is significant only for a BMI of 30 kg/m2 . Office spirometry reveals a decreased FVC and a normal FEV1/FVC ratio, and there are no changes after bronchodilator administration. Which one of the following would you recommend at this point? A) The 6-minute walk test B) Bronchoprovocation testing such as a methacholine challenge test C) Full pulmonary function testing D) Bronchoscopy E) A ventilation-perfusion scan

C The American Thoracic Society recommends full pulmonary function testing when office spirometry suggests a restrictive pattern, which is the case with this patient's normal FEV1/FVC ratio and decreased FVC.

Which one of the following is needed to calculate the number needed to treat (NNT)? A) Number needed to harm B) Pretest probability C) Absolute risk reduction D) Relative risk reduction E) Likelihood ratio

C The number needed to treat (NNT) is calculated as: 1/absolute risk reduction (ARR), where the ARR is expressed as a decimal.

A 42-year-old female presents with a several-month history of fatigue, arthralgias in her knees and hips, myalgias, hair loss, and a recent episode of gross hematuria diagnosed at an urgent care center as a urinary tract infection. She has no urinary tract symptoms at this time. A friend of hers who had similar symptoms for months was recently diagnosed with systemic lupus erythematosus (SLE), and the patient asks whether she might have SLE. Which one of the following would be most helpful in reassuring her that the likelihood of her having SLE is low? A) Absence of the typical malar rash B) Absence of RBC casts on microscopic urinalysis C) A negative serum antinuclear antibody D) Normal levels of complement C3, C4, and CH50 E) Joint pain limited to large joints

C The vast majority (>95%) of patients with SLE have a positive antinuclear antibody (ANA) test, thus it is sensitive as an initial test in a patient for whom there is clinical suspicion for SLE

An 86-year-old male is brought to your office by his daughter for follow-up of hypertension. His blood pressure is well controlled on amlodipine (Norvasc), 5 mg daily. His daughter is concerned about the safety of her father driving because he has become confused on several occasions. He tells you that he only drives short distances to familiar places during the daytime, and he is somewhat agitated that his daughter mentioned this topic. He has no cognitive deficits noted on a 3-item recall or a clock drawing test. Neurologic and musculoskeletal examinations reveal no deficits. Which one of the following would be the most appropriate next step in management? A) No further interventions B) Discontinuing amlodipine C) Reinforcing safe driving practices with education on age-related changes that may affect safe driving D) Recommending that he stop driving and surrender his license

C This patient is over the age of 85 and at higher risk for a motor vehicle crash. He has shown that he has good insight and has made safety changes to his driving. Reinforcing safe driving practices would be appropriate at this time.

A 34-year-old female presents with a 1-month history of increasing foot pain. She does not have any significant past musculoskeletal history, and she started a new exercise program 6 weeks ago. She has pain in the lateral side of her heel that is present both with activity and at rest. On examination you note tenderness below the lateral malleolus extending to the midfoot. Which one of the following is the most likely diagnosis? A) Calcaneal apophysitis (Sever's disease) B) Calcaneal stress fracture C) Peroneal tendinopathy D) Plantar fasciitis E) Tarsal tunnel syndrome

C This patient most likely has peroneal tendinopathy, which is a degeneration of the peroneal tendon that involves pain or tenderness in the lateral calcaneus below the ankle along the path to the base of the fifth metatarsal. Initial treatment options include activity modification, decreasing pressure to the affected area, anti-inflammatory or analgesic medications, and eccentric exercises. Calcaneal apophysitis, or Sever's disease, is a common growth-related injury that typically affects adolescents between 8 and 12 years of age. Symptoms often present after a growth spurt or starting a new high-impact sport or activity, and common examination findings include tight heel cords and a positive calcaneal squeeze test. A calcaneal stress fracture, which most commonly occurs immediately inferior and posterior to the posterior facet of the subtalar joint, involves pain that intensifies with activity and often worsens to include pain at rest.

A 42-year-old female sees you because of intermittent right upper abdominal pain that occurs after eating. The episodes have been gradually worsening and now last up to an hour. She has tried over-the-counter antacids, ibuprofen, and acetaminophen, which have not helped. She tells you that the last episode occurred earlier this week and the pain was so severe that it woke her up and she went to the emergency department (ED). A comprehensive metabolic panel, CBC, and lipase level performed in the ED were all normal. Right upper quadrant abdominal ultrasonography today is negative for gallstones but notable for increased echogenicity of the liver. Which one of the following would be the most appropriate next step in the evaluation? A) Plain radiography of the abdomen B) CT of the abdomen C) Hepatobiliary scintigraphy (HIDA) D) Magnetic resonance cholangiopancreatography (MRCP) E) Endoscopic retrograde cholangiopancreatography (ERCP)

C This patient presents with classic biliary symptoms and normal right upper quadrant ultrasonography, liver enzymes, and pancreatic enzymes. Abdominal ultrasonography was negative for gallstones. The next most appropriate test is hepatobiliary scintigraphy, also known as a hepatobiliary iminodiacetic acid (HIDA) scan. While a normal HIDA scan does not exclude a diagnosis of functional gallbladder disease (also referred to as acalculous cholecystitis, biliary dyskinesia, and biliary dysmotility), an abnormal study identifies patients for whom cholecystectomy is strongly recommended.

A 60-year-old male who is a bricklayer presents to your office in Florida with a fever, fatigue, headaches, night sweats, cough, and intermittent dyspnea. He also has myalgias and arthralgias. His symptoms started after he returned from a job in Arizona 2 weeks ago. He does not have any other travel history or sick contacts. His vital signs and a physical examination are unremarkable. A chest radiograph does not show any acute pathology. A CBC shows eosinophilia and his erythrocyte sedimentation rate is mildly elevated. Which one of the following is the most likely pathogen? A) Aspergillus B) Blastomyces C) Coccidioides D) Cryptococcus E) Histoplasma

C This patient traveled to an endemic area and engaged in dusty outdoor activities, which puts him at a higher risk for infection, and he presents with common symptoms of primary pulmonary coccidioidomycosis. A chest radiograph often appears normal on the initial evaluation. Eosinophilia should raise suspicion for coccidioidomycosis but laboratory detection of Coccidioides is required for a definitive diagnosis

A 20-year-old male presents with a painful second finger after his right hand was stepped on 3 days ago while he was playing basketball. He has marked pain as well as numbness of the distal finger. There are no open wounds and the skin color and nail appear normal other than moderate edema of the fingertip. A radiograph reveals a distal phalanx fracture. Which one of the following would be the most appropriate next step? A) Treat symptomatically with ice and an anti-inflammatory medication B) Tape the first and second digits together until symptoms resolve C) Splint the affected digit for 2-4 weeks D) Remove the nail to evaluate for a nail bed injury E) Refer to a hand surgeon

C Tuft fractures are the most common type of distal phalanx fracture, often result in up to 6 months of hyperesthesia, pain, and numbness. Tx involves splinting the affected digit for 2-4 weeks, followed by ROM and strengthening exercises.

A 70-year-old female sees you for a Medicare annual wellness visit. Her past medical history includes hypertension treated with enalapril (Vasotec). She states that she "couldn't be better" and says that she has no new symptoms or health concerns. She has a blood pressure of 159/90 mm Hg, a temperature of 36.7°C (98.1°F), a heart rate of 76 beats/min, a respiratory rate of 17/min, and an oxygen saturation of 98% on room air. On examination you note a new harsh systolic murmur that is heard best at the second right intercostal space and can also be heard over the right carotid artery. A transthoracic echocardiogram reveals severe aortic stenosis. Which one of the following should you recommend for this patient? A) Antibiotic prophylaxis for dental procedures B) Transesophageal echocardiography C) Repeat echocardiography in 6 months D) Referral for aortic valve replacement

C Watchful waiting is recommended for most asymptomatic patients. In asymptomatic patients with severe aortic stenosis, monitoring with serial echocardiography is recommended every 6-12 months. Antibiotic prophylaxis is not indicated unless the patient has undergone aortic valve replacement or has a history of endocarditis.

A 68-year-old male with a history of COPD, hypertension, and hyperlipidemia presents with a worsening cough and dyspnea with exertion over the past 3 months. His symptoms were previously well controlled with tiotropium (Spiriva) daily and albuterol (Proventil, Ventolin) as needed, and he has not had any COPD exacerbations in the past year until these symptoms began. He has not had any change in sputum production. Recently he has been using his albuterol inhaler several times a day to help relieve his shortness of breath with exertion. A physical examination reveals a temperature of 37.0°C (98.6°F), a heart rate of 78 beats/min, a respiratory rate of 16/min, a blood pressure of 144/82 mm Hg, and an oxygen saturation of 95% on room air. A cardiac evaluation reveals a regular rate and rhythm and he has no peripheral edema or cyanosis. His lungs are clear with no wheezes or crackles, and there is a mild prolonged expiratory phase. According to current GOLD guidelines, which one of the following would be the most appropriate next step in the management of this patient's symptoms? A) Add azithromycin (Zithromax) B) Add inhaled fluticasone (Flovent) C) Add inhaled salmeterol (Serevent) D) Add inhaled fluticasone/salmeterol (Advair) E) Discontinue tiotropium and start inhaled fluticasone

C n patients on monotherapy with a long-acting bronchodilator such as a long-acting muscarinic agonist (LAMA) or long-acting -agonist (LABA) who have continued dyspnea, the Global Initiative for Chronic Obstructive Lung Disease (GOLD) guidelines recommend escalating therapy to two bronchodilators.

A 43-year-old female comes to your clinic for a routine health maintenance examination. She has a past medical history of diarrhea-predominant irritable bowel syndrome (IBS-D), recurrent urinary tract infections (UTIs), and bacterial vaginosis. She has no new health concerns today. She does not take any medications on a regular basis, and states that she prefers natural supplements to prescription medications. She says that she has heard that oral probiotics are beneficial and asks if they might be the right choice for her. Which one of the following is the best evidence-based approach to counseling her about oral probiotics? A) There is no evidence that they will improve her IBS B) There is no evidence that they will decrease the risk of Clostridioides (Clostridium) difficile diarrhea when she is treated for a UTI C) There is strong evidence that they will decrease the risk of antibiotic-associated diarrhea when she is treated for a UTI D) There is strong evidence that they will decrease the risk of UTI recurrence E) There is strong evidence that they will decrease the risk of bacterial vaginosis recurrence

C there is strong evidence based on Cochrane reviews that the use of probiotics may reduce the risk of both antibiotic-associated diarrhea more generally, and Clostridioides (Clostridium) difficile diarrhea specifically, when antibiotics are used (level of evidence A). Evidence is not as strong for their impact in adults over the age of 65.

A 42-year-old female presents to your office to discuss bariatric surgery and its potential complications. Her BMI is 40 kg/m2 and she has hypertension, type 2 diabetes, and osteoarthritis of both knees. If she opts to have a sleeve gastrectomy, which one of the following complications is most likely in the first 6 months? A) Cholelithiasis B) Dumping syndrome C) GERD D) Leaking at the surgical site E) Small bowel obstruction

C The most common complication is development of GERD, which occurs in 20% of patients. Since this procedure does not produce a malabsorption component, complications such as cholelithiasis, dumping syndrome, and small bowel obstruction are not as likely as with other available procedures

An otherwise healthy 58-year-old female presents with several noduloulcerative lesions ascending up her arm from a lesion on her index finger. A fungal culture of drainage from one of the lesions reveals Sporothrix schenckii. Which one of the following would be the most appropriate initial management strategy? A) Observation for spontaneous resolution for 1 month B) Local application of heat C) Oral itraconazole (Onmel) D) Oral saturated solution of potassium iodide E) Intravenous liposomal amphotericin B (AmBisome)

C Uncomplicated small lesions of cutaneous sporotrichosis sometimes can be treated with the daily application of local heat for several weeks. More involved infections, such as this patient's lymphocutaneous sporotrichosis, require systemic therapy. The initial treatment strategy is oral itraconazole for 3-6 months.

A 45-year-old female with a 4-year history of type 2 diabetes is taking only metformin (Glucophage) and maintaining a hemoglobin A1c of 6.6%. Her LDL-cholesterol level is 94 mg/dL. She has no complications related to diabetes and her medical history is otherwise unremarkable. Which one of the following should be added to her current medication regimen? A) A DPP-4 inhibitor B) An SGLT2 inhibitor C) A low-intensity statin D) A moderate-intensity statin E) A high-intensity statin

D All patients between 40 and 75 years of age with diabetes mellitus and an LDL-cholesterol level ≥70 mg/dL should begin taking a moderate-intensity statin. It is not necessary to calculate a 10-year risk for atherosclerotic cardiovascular disease because the results do not alter the recommendation.

Which one of the following antihypertensive medications is LEAST likely to exacerbate erectile dysfunction? A) Clonidine (Catapres) B) Doxazosin (Cardura) C) Hydrochlorothiazide D) Losartan (Cozaar) E) Metoprolol

D Angiotensin receptor blockers (ARBs) such as losartan are least likely to cause or exacerbate erectile dysfunction. ARBs may have a favorable effect on erectile dysfunction by inhibiting vasoconstriction activity of angiotensin. Clonidine, -blockers, hydrochlorothiazide, and -blockers are more likely to negatively affect erectile function

A 45-year-old female comes to your office for an annual health maintenance visit. She has a family history of type 2 diabetes in her mother and a personal history of obesity for many years. If verified with a second test, which one of the following would confirm a diagnosis of diabetes mellitus? A) A hemoglobin A1c of 6.4% B) A 2-hour plasma glucose level of 190 mg/dL on an oral glucose tolerance test C) A random glucose level of 190 mg/dL D) A fasting plasma glucose level of 130 mg/dL

D A diagnosis of type 2 diabetes can be based on any of the following test results: a hemoglobin A1c ≥6.5%, a fasting plasma glucose level ≥126 mg/dL, a 2-hour plasma glucose level ≥200 mg/dL on an oral glucose tolerance test, or a random plasma glucose level ≥200 mg/dL with classic symptoms of hyperglycemia

A 55-year-old female with diabetes mellitus and hypertension sees you because of a 3-month history of a persistent nonproductive cough. Two weeks after the cough began she presented to a local urgent care center with additional symptoms of sinus pressure, rhinorrhea, and subjective wheezing. A lung examination and chest radiograph performed at that visit were unremarkable. She was diagnosed with acute bronchitis and prescribed benzonatate (Tessalon). Since then, her sinus-related symptoms have abated, although her cough has not improved. Her current medications include metformin (Glucophage), lisinopril (Prinivil, Zestril), and hydrochlorothiazide, all of which were initiated 6 months ago. She has no known allergies and has never smoked. A physical examination today is unremarkable. Which one of the following is the most likely cause of her cough? A) Chronic lung disease B) Infection C) Malignancy D) A medication side effect E) A psychogenic habit

D ACE-I make chronic coughs. Psychogenic cough is rare.

A 53-year-old female with diabetes mellitus presents to the emergency department with a 4-day history of nausea and vomiting. Her vital signs include a blood pressure of 142/93 mm Hg, a temperature of 36.5°C (97.7°F), a heart rate of 93 beats/min, a respiratory rate of 18/min, and an oxygen saturation of 98% on room air. A potassium level is 6.8 mEq/L (N 3.5-5.1) and a fingerstick blood glucose level is 120 mg/dL. The patient has chronic kidney disease with a baseline glomerular filtration rate of 32 mL/min/1.73 m2 and today it is 18 mL/min/1.73 m2 . Which one of the following EKG findings would you expect? A) Atrial fibrillation B) Diffuse ST-segment elevation C) Peaked P waves D) Peaked T waves E) Sinus tachycardia

D Acute EKG changes may be noted in the setting of hyperkalemia. These changes should trigger prompt treatment of the electrolyte abnormality, but it should be noted that they are nonspecific in nature and treatment should not be solely based on these findings. EKG changes noted with hyperkalemia include peaked T waves, flattened P waves, PR prolongation, a widened QRS complex, sine waves, sinus bradycardia, ventricular tachycardia, ventricular fibrillation, and asystole.

An 80-year-old male is considering a cardiac procedure. As part of the evaluation for the procedure he is found to meet the criteria for being at risk of frailty (prefrail). Which one of the following would be the most effective management of this patient's prefrail status? A) Nutritional supplementation B) Vitamin D supplementation C) Testosterone supplementation D) A physical activity program E) Cognitive-behavioral therapy

D Adults who are diagnosed as either prefrail or frail should be considered for a multicomponent physical activity program.

A 50-year-old female sees you for further evaluation after she had elevated blood pressure on two consecutive visits to an urgent care clinic for minor illnesses. She has no history of chest pain, shortness of breath, headache, or changes in vision. She has smoked half a pack of cigarettes daily since the age of 16 and has an allergy to iodine. There is no other significant medical history. On examination the patient has a blood pressure of 155/92 mm Hg and a heart rate of 80 beats/min. The cardiovascular and pulmonary examinations are otherwise normal. A basic metabolic panel and a TSH level are normal. In addition to lifestyle changes, which one of the following would be most appropriate as initial pharmacologic therapy for management of this patient's hypertension? A) Carvedilol B) Furosemide (Lasix) C) Hydralazine D) Hydrochlorothiazide E) Spironolactone (Aldactone)

D Appropriate initial treatment of hypertension should include a thiazide-type diuretic, calcium channel blocker, ACE inhibitor, or angiotensin receptor blocker.

A 69-year-old male sees you for a routine examination and asks about lung cancer screening. He smoked one pack of cigarettes per day for about 35 years but quit 11 years ago. According to the U.S. Preventive Services Task Force and the American College of Chest Physicians, which one of the following should you recommend? A) No screening B) An annual history and examination focusing on lung symptoms C) Annual chest radiography D) Annual low-dose chest CT

D CT scan of chest in patients 50-80 with > 20 pk year history who are current smokers or who have smoked in the past 15 years

In a patient diagnosed with major depressive disorder, which one of the following factors should raise suspicion for a bipolar disorder? A) Diminished interest in almost all activities B) Fatigue or loss of energy C) Feelings of worthlessness or inappropriate guilt D) A parent diagnosed with a bipolar disorder E) Psychomotor retardation

D Children of parents with bipolar disorders have a 4%-15% risk of being affected compared to <2% of patients without a family history

A 14-year-old male is brought to your office by his parents, who are concerned about his behavior. Recently he was caught shoplifting video games. He started smoking cigarettes at age 10, and he has a history of truancy from school for the past 2 years. His parents report that they have caught him starting fires outside of their home, and he often teases the family dog, whom he has injured on several occasions. The most likely diagnosis is A) antisocial personality disorder B) attention-deficit/hyperactivity disorder (ADHD) C) avoidant personality disorder D) conduct disorder E) substance abuse

D Conduct disorder most commonly occurs during adolescence and childhood. There are multiple criteria, including aggression toward people and animals, theft, starting fires, and truancy.

A 67-year-old female who is a retired teacher presents with generalized itching. She tells you that she is convinced that she has acquired a skin infestation from small mites. She gives you a matchbox containing what appears to be crusts, dried blood, and bits of skin as evidence of this problem. A previous physician had obtained a CBC, comprehensive metabolic panel, TSH level, chest radiograph, and drug screen, which were all normal. An examination today reveals excoriations on her arms, abdomen, and legs in easily reached areas. Her skin is not dry and there are no lesions in her axillae or web spaces. Which one of the following medications would be most likely to help this patient? A) Cholestyramine (Questran) B) Hydroxyzine (Vistaril) C) Ivermectin (Stromectol) D) Olanzapine (Zyprexa) E) Prednisone

D Delusion of infestation - rule out organic causes first. Most patients are female and either retired or disabled. Patients often respond to atypical antipsychotic meds such as risperidone or olanzapine. Cholestyramine is used to treat cholestatic jaundice. Hydroxyzine can be used for itching, particularly from urticaria, but can cause sedation in the elderly. Ivermectin is an option to treat scabies. Prednisone would be appropriate for allergic reactions or inflammatory dermatitis problems.

A 44-year-old female sees you for evaluation of frequent, disabling headaches. She describes the headaches as throbbing, usually left-sided, and associated with nausea. Her headaches worsen with sound and movement, and they last for 48 hours nearly every week. She has not had any aura, weakness, numbness, or speech disturbance. Based on current evidence, which one of the following medications would have the greatest potential for reducing the number of headache days for this patient? A) Fluoxetine (Prozac) B) Gabapentin (Neurontin) C) Lisinopril (Prinivil, Zestril) D) Propranolol E) Verapamil (Calan)

D Divalproex, topiramate, metoprolol, propranolol, and timolol have been shown to be effective for migraine prevention by consistent, good-quality evidence.

An obese 32-year-old male is admitted to the hospital with a new onset of acute pancreatitis. A lipid panel reveals a triglyceride level of 1150 mg/dL and an HDL-cholesterol level of 30 mg/dL. Other laboratory studies are normal. His 10-year risk of atherosclerotic cardiovascular disease is <5%. His family history is positive for recurrent pancreatitis in his father and paternal grandfather. In addition to lifestyle modifications, which one of the following would be most appropriate for this patient after he is discharged? A) Atorvastatin (Lipitor) B) Colesevelam (Welchol) C) Ezetimibe (Zetia) D) Fenofibrate (Tricor) E) Omega-3-acid ethyl esters (Lovaza)

D Fibrates reduce the likelihood and recurrence of pancreatitis due to severe hypertriglyceridemia when triglyceride levels are ≥500 mg/dL, measured in a fasting or nonfasting state

A 3-year-old male has developed multiple large areas of bullous impetigo on the legs, buttocks, and trunk after being bitten numerous times by ants. Which one of the following would be the most appropriate treatment? A) Topical mupirocin ointment B) Oral azithromycin (Zithromax) C) Oral tetracycline D) Oral trimethoprim/sulfamethoxazole (Bactrim) E) Intramuscular penicillin G benzathine (Bicillin L-A)

D Impetigo may be caused by Streptococcus pyogenes or Staphylococcus aureus, but bullous impetigo is caused exclusively by S. aureus

After a thorough history and examination you determine that a 30-year-old male has an upper respiratory infection with a persistent cough. He is afebrile and is otherwise healthy. The best treatment for symptomatic relief of his persistent cough would be intranasal A) antibiotics B) antihistamines C) corticosteroids D) ipratropium (Atrovent) E) saline

D Intranasal ipratropium is the only medication that improves persistent cough related to upper respiratory infection in adults. Intranasal antibiotics, antihistamines, corticosteroids, and saline would not improve this patient's cough.

A 59-year-old male presents with difficulty breathing during exercise. He says that his symptoms have gradually worsened over the past year and he has had to discontinue his morning walks. He reports mild lower extremity edema and weight gain. He has a blood pressure of 115/79 mm Hg, a heart rate of 88 beats/min, and an oxygen saturation of 92% on room air. A physical examination is notable for mild jugular venous distention and 1+ bilateral lower extremity edema. Examination of the heart reveals a normal rate and rhythm with an S3 heart sound. The lungs are clear to auscultation. You order a CBC, a comprehensive metabolic panel, an EKG, a chest radiograph, and echocardiography. While awaiting the results you consider the differential diagnosis. Which one of the following conditions is the most common cause of pulmonary hypertension? A) Chronic thromboembolism B) COPD C) Idiopathic pulmonary arterial hypertension D) Left heart disease E) Sleep-disordered breathing

D Left heart disease, including both preserved and reduced systolic function, is the most common cause of pulmonary hypertension, while chronic thromboembolism, COPD, and sleep-disordered breathing are other possible but less common causes.

An 18-month-old female is brought to your office for a well child check. During the examination you note that she is unable to say any words. She can follow a one-step command and point to three body parts. She does wave goodbye and babble, and she appears to have normal comprehension, emotional relationships, and fine and gross motor movements. Which one of the following is the most likely diagnosis based on this patient's speech development? A) Autism spectrum disorder B) Cerebral palsy C) Congenital hearing loss D) Developmental speech and language delay E) Receptive language disorder

D More than 90% of children can speak three words at 18 months of age and 50%-90% can speak six words. This patient scenario suggests a developmental speech delay.

A 46-year-old male comes to your office for a routine health maintenance examination. He had elevated AST and ALT levels on previous laboratory testing. He reports consuming only one alcoholic drink twice a month. He does not take any medications, including over-the-counter medications. A physical examination is normal except for a BMI of 32 kg/m2 . Laboratory testing shows an AST level of 72 U/L (N 10-59) and an ALT level of 96 U/L (N 13-40). A hepatitis panel, serum ferritin level, serum iron level, lipid profile, and fasting glucose level are all within normal limits. Ultrasonography of the right upper quadrant shows fatty infiltration of the liver. Which one of the following would be the first-line treatment of this condition? A) Biguanides such as metformin (Glucophage) B) GLP-1 analogs such as liraglutide (Victoza) C) Thiazolidinediones such as pioglitazone (Actos) D) Weight loss with diet and exercise E) Bariatric surgery

D Obesity is a risk factor for NAFLD, and the primary treatment of NAFLD is weight loss with diet and exercise.

A 22-year-old gravida 1 para 0 was recently diagnosed with gestational diabetes. Her fasting blood glucose levels have consistently been 120-130 mg/dL since she began following nutrition and exercise recommendations. Based on guidelines from the American Diabetes Association and the American College of Obstetricians and Gynecologists, which one of the following would be preferred at this point? A) No change in management B) Loosening the calorie restrictions C) Adding glyburide D) Adding insulin E) Adding metformin (Glucophage)

D Pharmacologic tx should be initiated in patients with GDM when nutrition and exercise are not adequate. Goals are fasting BG < 95, 1 hour postprandial < 140 and 2-hour glucose levels <120. Insulin is preferred treatment in GDM. Oral meds can be started in patients who refuse insulin or are unable to comply.

A 62-year-old male with hypertension and metabolic syndrome sees you for follow-up. A fasting triglyceride level is 300 mg/dL. You address lifestyle and other potential causes of his elevated triglycerides, including his current medications. If included in his current regimen, which one of the following hypertension medications would be most likely to contribute to his hypertriglyceridemia? A) Amlodipine (Norvasc) B) Diltiazem (Cardizem) C) Lisinopril (Prinivil, Zestril) D) Metoprolol

D Several medications can be secondary causes of hypertriglyceridemia, including -blockers, with the exception of carvedilol. Others include oral estrogens, glucocorticoids, bile acid sequestrants, protease inhibitors, retinoic acid, anabolic steroids, sirolimus, raloxifene, tamoxifen, and thiazides.

An 18-month-old female develops immune thrombocytopenic purpura following MMR administration in your clinic. She requires hospitalization and extensive treatment. The child does not have any health insurance coverage. The child's family could be eligible for compensation by the A) clinic's malpractice insurance B) vaccine manufacturer's liability coverage C) Countermeasures Injury Compensation Program D) National Vaccine Injury Compensation Program

D The National Childhood Vaccine Injury Act of 1986 established the no-fault National Vaccine Injury Compensation Program for patients and families who were injured by recommended vaccines.

A 34-year-old male began a sexual relationship with a woman 3 months ago and the relationship ended on friendly terms last week. He received a call yesterday from the woman, who said she had developed a rash that resulted in a diagnosis of syphilis and that he should be evaluated and treated if appropriate. He has no symptoms and a serologic test for syphilis is negative. He has no known drug allergies. Which one of the following would be most appropriate at this time? A) Daily self-inspection of the penis to identify a chancre B) Observation with a repeat serologic test for syphilis in 6 weeks C) Azithromycin (Zithromax), 2 g orally as a single dose D) Penicillin G benzathine (Bicillin L-A), 2.4 million units intramuscularly as a single dose E) Penicillin G benzathine, 2.4 million units intramuscularly once weekly for 3 weeks

D The evidence suggests that this patient did not have syphilis prior to this lone contact and a diagnosis of syphilis cannot be confirmed by examination or testing at this point. He should be treated presumptively for early syphilis, even though the serologic test result is negative, because he had sexual contact within the past 90 days with a person who was diagnosed with secondary syphilis.

You are reviewing and updating your routine health care examination electronic health record templates to include formal recommendations from the U.S. Preventive Services Task Force. You also consider age-specific causes of mortality in order to create corresponding preventive strategies. Which one of the following is the leading cause of mortality among people 45-64 years of age? A) Accidents B) Diabetes mellitus C) Heart disease D) Malignancy E) Suicide

D The leading cause of mortality among people aged 45-64 years is malignancy. Accidents are 3rd most common COD in this age range, but 1st in people 15-44. DM is 6th most common COD in this age range Heart disease is 2nd most common COD in this age range, leading COD in adults >65yo

A 30-year-old female presents for evaluation of chronic abdominal bloating, cramping, diarrhea, and recent weight loss. An abdominal examination is unremarkable, and stool guaiac testing is negative. She requests testing for celiac disease. Which one of the following would be most likely to cause a false-negative result on serologic testing for celiac disease? A) A recent increase in dietary wheat consumption B) Recent use of loperamide (Imodium A-D) C) A skin rash consistent with dermatitis herpetiformis D) IgA deficiency E) Iron deficiency anemia

D The preferred initial diagnostic test includes a serum IgA transglutaminase-2 (TG2) antibody level, which has a 98% sensitivity and 98% specificity for the diagnosis of celiac disease. False-negative serologic results may occur in patients with an IgA deficiency, which includes up to 3% of patients with celiac disease. Therefore, when a diagnosis of celiac disease is strongly suspected despite a negative IgA TG2 antibody test, a total IgA level should be obtained

A 50-year-old male with newly diagnosed type 2 diabetes asks how to reduce his risk of diabetic retinopathy progression. You tell him that in addition to maintaining good glycemic control, the risk can be reduced by A) corticosteroid eye drops B) an ACE inhibitor C) aspirin therapy D) blood pressure control E) lipid management

D The risk of diabetic retinopathy progression can be modified by good glycemic control, maintaining a hemoglobin A1c <7%, maintaining a blood pressure <140/90 mm Hg, and undergoing periodic eye examinations

You practice in a community with a significant number of Laotian immigrants. A 53-year-old male comes to your office accompanied by his adult daughter for follow-up of hypertension. His primary language is Lao and he has limited proficiency in English. According to Title VI of the Civil Rights Act, which one of the following is the correct way to approach this situation? A) Continue without an interpreter unless he asks for one B) Ask the patient's daughter to interpret C) Ask a nurse who speaks Lao to interpret D) Offer a trained interpreter E) Speak loudly and slowly and give him written instructions

D This one is obvious

A premenopausal 48-year-old female comes to your office because of a 3-month history of increased frequency and volume of vaginal bleeding. She typically menstruates every 24 days but now has bleeding every 10-14 days and notes an increased volume of blood as well. She feels well otherwise. Her cervical cancer screening is current. A bimanual examination reveals blood at the cervical os. Otherwise the cervix appears normal, the uterus is normal to palpation, and there are no adnexal masses. Which one of the following should be the next step in the evaluation? A) Reassurance that irregular menses are common in the perimenopausal period B) HPV testing C) CT of the pelvis with and without contrast D) An endometrial biopsy

D This patient has abnormal uterine bleeding characterized by an increased frequency and volume of vaginal bleeding. Due to the increased risk of endometrial cancer, current guidelines recommend that all women >45 years of age presenting with abnormal uterine bleeding undergo endometrial sampling. Irregular menses can occur during the perimenopausal period but this patient's increased frequency and volume of vaginal bleeding combined with her age warrant further evaluation.

A 34-year-old male sees you via your clinic's electronic portal because of a rash. The rash, which he first noticed 3 days ago, was a large red patch on his upper leg at that time. He uploads an image of the rash (bulls eye) as it appears today. He started feeling feverish last night with chills, nausea, headache, and fatigue. He lives in Wisconsin and spends much of his free time hiking in the woods near his home. He removed two ticks from his legs last week. Which one of the following is the most likely cause of his current symptoms? A) Anaplasmosis B) Babesiosis C) Ehrlichiosis D) Lyme disease E) Tularemia

D This patient has findings consistent with early localized Lyme disease, notably influenza-like symptoms and an erythema migrans (EM) rash with its typical bull's-eye or target-like appearance. It is the most common tickborne disease in the United States, and it is most prevalent in states in the New England, mid-Atlantic, and upper Midwest regions.

A 28-year-old female presents with a depressed mood and sleep disturbance. She tells you that this has occurred for the past 4 years but only during the winter months. Her past medical history and a physical examination are unremarkable. Which one of the following interventions has the strongest evidence for preventing recurrence of her condition? A) Exercise B) Light therapy C) Cognitive-behavioral therapy D) Bupropion (Wellbutrin XL) E) Fluoxetine (Prozac)

D This patient has seasonal affective disorder (SAD) that has recurred and is likely to continue to recur. Bupropion is the only medication beneficial for prevention of SAD. Light therapy and SSRIs are helpful for treating this disorder but do not prevent it. Exercise and cognitive-behavioral therapy are beneficial adjuncts to treatment but would not prevent recurrence.

A 65-year-old female sees you because of increased irritability and confusion. She has a history of major depression, essential hypertension, and type 2 diabetes. Her medications include sertraline (Zoloft), 100 mg daily; lisinopril (Prinivil, Zestril), 20 mg daily; and metformin (Glucophage), 500 mg twice daily. She recently sustained a right distal radius fracture as a result of a fall, and she has been taking tramadol, 50 mg, every 6 hours for pain control. On examination the patient has a temperature of 38.2°C (100.8°F), a pulse rate of 96 beats/min, a respiratory rate of 16/min, and a blood pressure of 124/78 mm Hg. She appears confused and you note a bilateral tremor in the upper extremities, brisk reflexes, and two beats of clonus on a bilateral foot examination. The examination is otherwise normal. A CBC, comprehensive metabolic panel, chest radiograph, and CT of the head are all within normal limits. Which one of the following is the most likely cause of this patient's symptoms? A) Acute cerebral infarction B) Meningitis C) Sepsis syndrome D) Serotonin syndrome

D This patient has serotonin syndrome based on her use of both sertraline and tramadol and meeting the Hunter criteria of hyperthermia, clonus, tremor, and hyperreflexia.

A 24-year-old female comes to your office with a 1-day history of the gradual onset of nonradiating worsening right lower quadrant abdominal pain, nausea, emesis, and chills. Her symptoms are minimally relieved with ibuprofen, 400 mg. Her menstrual period 2 weeks ago was normal. She has not had any dysuria, vaginal discharge, or change in bowel habits. Her past medical and surgical histories are notable only for dysmenorrhea managed with an anti-inflammatory medication as needed. An examination reveals a temperature of 38.1°C (100.6°F), discomfort with position change, right lower quadrant abdominal tenderness and guarding, and diminished bowel sounds. Point-of-care testing shows an elevated WBC count, and a urinalysis and pregnancy test are negative. Which one of the following would you recommend at this time? A) A plain film of the abdomen B) Abdominal ultrasonography C) Pelvic ultrasonography with transvaginal ultrasonography if needed D) CT of the abdomen and pelvis E) MRI of the abdomen and pelvis

D This patient has suspected appendicitis, and CT of the abdomen and pelvis with intravenous contrast is the preferred initial imaging study. Ultrasonography is preferred in children, but not adults, as the initial study for suspected appendicitis.

A 23-year-old gravida 1 para 1 who is a single mother of a 3-day-old infant comes to your office for a newborn follow-up. She reports some sleep disturbance, mild depression without suicidal ideation, and financial concerns. Her past medical history is significant for persistent depressive disorder. The U.S. Preventive Services Task Force recommends which one of the following to help prevent perinatal depression in patients such as this? A) Exercise B) Amitriptyline C) Sertraline (Zoloft) D) Referral for cognitive-behavioral therapy

D USPSTF recommends counseling interventions to prevent perinatal depression in patients who are at risk.

A 68-year-old female comes to your office for a follow-up visit for diabetes mellitus. Her home glucose monitor record shows a range of 68-125 mg/dL. Her medications include atorvastatin (Lipitor), 40 mg daily; metformin (Glucophage), 750 mg twice daily; and insulin glargine (Lantus), 10 U nightly. Laboratory studies are remarkable for a hemoglobin A1c of 5.8% and a creatinine level of 0.98 mg/dL (N 0.6-1.1). She maintains healthy lifestyle behaviors such as walking 30 minutes 5 days per week and avoiding sweetened beverages. Which one of the following would be the most appropriate treatment plan? A) Continue the current medication regimen B) Increase the insulin glargine dosage C) Increase the metformin dosage D) Discontinue insulin glargine E) Discontinue metformin

D Well controlled, hypoglycemia - stop insulin

Which one of the following psychoactive medications would create the greatest risk of respiratory depression if used in combination with an opioid? A) Amitriptyline B) Bupropion (Wellbutrin) C) Escitalopram (Lexapro) D) Lorazepam (Ativan) E) Trazodone

D clinicians should avoid prescribing opioid pain medication and benzodiazepines concurrently whenever possible

A 55-year-old male presents to your office to establish care. He has a history of hypertension, hypercholesterolemia, and coronary artery disease. He had a non-ST-elevation myocardial infarction (NSTEMI) 3 years ago. An echocardiogram at that time was normal and he received a single drug-eluting stent. He has not seen a cardiologist since then and would prefer not to see one unless it is necessary, due to his high insurance copayments for specialist visits. His current medications include clopidogrel (Plavix), aspirin, atorvastatin (Lipitor), lisinopril (Prinivil, Zestril), and carvedilol (Coreg). A review of systems is negative. His vital signs include a blood pressure of 120/72 mm Hg, a heart rate of 80 beats/min, and a respiratory rate of 12/min. A physical examination is unremarkable, a basic metabolic panel is normal, and his hemoglobin A1c is 5.7%. His LDL-cholesterol level is 60 mg/dL, his HDL-cholesterol level is 49 mg/dL, and his total cholesterol level is 147 mg/dL. The patient would like to reduce the number of medications he takes because of the cost. Which one of the following medications in his current regimen would be most appropriate to discontinue at this time? A) Aspirin B) Atorvastatin C) Carvedilol D) Clopidogrel E) None of his current medications

D it would be reasonable for this patient to stop dual antiplatelet therapy at this time by discontinuing clopidogrel. Aspirin should be continued and is the most cost-effective option for antiplatelet therapy.

A 33-year-old female presents with palpitations and excessive sweating. A physical examination is normal. Laboratory findings include a TSH level of 0.02 U/mL (N 0.40-4.00) and a free T4 level of 3.9 ng/dL (N 0.7-1.9). Radionuclide scanning reveals no uptake. Which one of the following would explain these findings? A) Thyroid hormone resistance B) Graves disease C) A toxic nodular goiter D) Excess thyroid hormone intake E) A thyrotropin-secreting pituitary tumor

D Excess thyroid hormone intake would cause a low TSH level with a high free T4 level. Other possibilities include an hCG-secreting tumor and the thyrotoxic phase of subacute thyroiditis An elevated TSH level would be seen with thyroid-hormone resistance or a thyrotropin-secreting pituitary tumor. Graves disease causes a homogeneous increased thyroid uptake on radionuclide scanning, whereas a hot nodule would be expected with toxic nodular goiter.

A 56-year-old male who has heart failure with reduced ejection fraction sees you for follow-up. He is stable but over the past year has noted an increase in dyspnea with moderate activity. His blood pressure is well controlled today. His current medications include carvedilol (Coreg), losartan (Cozaar), and escitalopram (Lexapro). Which one of the following additions to his current medication regimen has the best evidence for reducing his risk of mortality from heart failure? A) Aspirin B) Atorvastatin (Lipitor) C) Furosemide (Lasix) D) Hydrochlorothiazide E) Spironolactone (Aldactone)

E Both -blockers and aldosterone antagonists have been shown to reduce mortality in patients with symptomatic heart failure

You admit a 68-year-old female with an acute stroke to the hospital. She has no other acute cardiovascular conditions. CT rules out a hemorrhagic event. You have determined that the patient is not a candidate for reperfusion therapy with alteplase or thrombectomy. You advise the nursing staff that you will be initiating antihypertensive therapy if the patient's blood pressure rises above a threshold of A) 120/80 mm Hg B) 140/90 mm Hg C) 160/100 mm Hg D) 180/110 mm Hg E) 220/120 mm Hg

E Because patients with an acute ischemic stroke may require the increased perfusion pressure to limit ischemia, antihypertensive therapy should not be given during the first 48-72 hours as long as they are not candidates for, or recipients of, reperfusion therapy with alteplase or thrombectomy; do not have a comorbid condition requiring acute blood pressure lowering; and do not have a blood pressure >220/120 mm Hg.

Which one of the following is the preferred method of diagnosing lymphoma in a 60-year-old male who presents with weight loss, unexplained fever, and axillary adenopathy? A) CT of the chest, pelvis, and abdomen pre- and post-contrast B) A PET-CT scan C) A bone scan D) A bone marrow aspiration and biopsy E) An open lymph node biopsy

E An open lymph node biopsy is the preferred method for making the diagnosis of lymphoma. Although fine-needle aspiration and core needle biopsy are often part of the initial evaluation of any adenopathy, neither will provide adequate tissue for the diagnosis of lymphoma. A PET-CT scan may be used for staging. A bone scan or CT alone is not part of the usual diagnostic evaluation.

A 55-year-old female presents with swelling and some redness in the area of her right ankle that had a gradual onset over the past week. She has not had any injury, fever, or other signs of systemic illness and has no pain. Her past medical history is significant for type 2 diabetes with polyneuropathy that is moderately well controlled, hypertension, hyperlipidemia, and a BMI of 35 kg/m2 . On examination her right ankle and foot are slightly larger than the left, exhibit faint erythema, and feel slightly warmer than the left. No pain is noted with palpation, and her ankle ligaments appear to be intact. Pedal pulses are 2+ bilaterally and she has no calf pain or swelling. Which one of the following would be the most appropriate next step? A) Reassure her that the lack of pain indicates the absence of a serious disease process B) Prescribe antibiotics for presumed cellulitis C) Recommend compression stockings, leg elevation, and monitoring D) Provide an ankle stabilizing brace E) Obtain bilateral weight-bearing foot radiographs

E Acute Charcot neuropathy is a commonly missed diagnosis, and the diagnosis is delayed in up to 25% of cases. The diagnosis should be considered in patients over age 40 with neuropathy and obesity who present with unilateral foot swelling. There may be associated erythema and warmth, and pain may be absent. In a patient with suspected acute Charcot neuropathy, bilateral weight-bearing radiographs are recommended to detect fractures of the midfoot.

A 57-year-old female with diabetes mellitus comes to your office for a routine follow-up. Her current medications include metformin (Glucophage), 1000 mg twice daily. She tells you that she does not exercise regularly and finds it difficult to follow a healthy diet. A hemoglobin A1c today is 7.5%. She does not want to add medications at this time, but she does want to get her hemoglobin A1c below 7%, which is the goal that was previously discussed. Which one of the following would be the most effective way to improve glucose control for this patient? A) Discuss the components of a healthy diabetic diet and encourage her to follow it more closely B) Discuss the importance of regular exercise and encourage her to exercise 30-45 minutes daily C) Recommend that she check her glucose level 1-3 times daily to help determine what adjustments need to be made D) Start her on an additional medication E) Refer her to a diabetes educator for medical nutrition therapy

E Counseling by a diabetic educator or team of educators for medical nutrition therapy lowers hemoglobin A1c by 0.2-0.8 percentage points in patients with type 2 diabetes.

A 14-year-old female is brought to your office for a well child check and a sports physical examination. During the substance abuse screening she says that she does not drink alcohol or smoke marijuana or traditional cigarettes, but occasionally uses e-cigarettes with her friends. Which one of the following statements describing e-cigarettes is true? A) E-cigarettes do not contain heavy metals B) The nicotine in e-cigarettes is not addictive C) Teens who use e-cigarettes are less likely to use marijuana D) More teens use traditional cigarettes than e-cigarettes E) E-cigarette use among teens increases the likelihood of cigarette smoking

E E-cigarette use has become quite popular among youth in the United States, with rates surpassing traditional cigarette use in 2014. Among teens who have never smoked, the odds of cigarette smoking are 3-6 times higher in those who have used e-cigarettes within the last year

A 69-year-old male presents with a several-week history of difficulty swallowing that has gradually worsened. At first he noted trouble with passing larger boluses of food through his mid-chest area. Now he states that even ice cream is a problem. He has a past history of GERD that he has treated with omeprazole (Prilosec) intermittently over the past several years. He also takes enteric-coated aspirin, 81 mg daily, for his heart. There is no history of tobacco use. He drinks about six beers a week. On examination you note a 5-kg (11-lb) weight loss over the past 3 months but the remainder of the examination is normal. Which one of the following would be the most appropriate next step in the evaluation? A) A trial of omeprazole, 40 mg daily, for 8 weeks B) Chest CT with and without contrast C) Barium esophagography D) High-resolution esophageal manometry E) Esophagogastroduodenoscopy

E Esophagogastroduodenoscopy (EGD) is needed without delay. This approach would allow biopsy of any lesions seen and therapeutic dilatation if a benign-appearing stricture is noted. Biopsies are also needed to diagnose eosinophilic esophagitis. If the EGD does not identify a problem, further workup should then proceed. CT may identify a source of extrinsic pressure. Barium esophagography may detect mild narrowing or esophageal webs missed on EGD

An exclusively breastfed 2-month-old male is brought to your office by his mother for a routine well child visit. His mother indicates that the infant received all recommended care at birth, is feeding well, and is meeting expected developmental milestones. Growth parameters are consistent with previously charted data. A physical examination is unremarkable except for bruising of the right lateral arm and left anterior thigh. When questioned about these findings, the mother states that she is not overly concerned, as he is "quite an active little kicker" when he is lying in his crib. She says that his 4-year-old and 6-year-old brothers are both physically active and bruise easily as well. Which one of the following would be most appropriate at this point? A) Reassurance that bruising of the extremities is common in this age group B) A serum vitamin D level C) A serum vitamin K level D) Abdominal ultrasonography E) Immediate referral for a child abuse investigation

E For the 2-month-old in this case, the presence of bruising on the arm and thigh should raise concern and prompt immediate referral for a child abuse investigation. For children under 2 years of age with suspected abuse, skeletal survey imaging is also recommended

A 48-year-old male with a 5-year history of diabetes mellitus comes to your office for a 6-month follow-up. His diabetes is finally under better control. His most recent hemoglobin A1c is 7.1% but his urine albumin excretion is 250 mg/24 hr (N <80), which is confirmed with two different readings. He is currently taking amlodipine (Norvasc), 5 mg daily, for blood pressure control. He previously took lisinopril (Prinivil, Zestril) but discontinued it after developing a bothersome dry cough. Which one of the following agents would be most appropriate to add to this patient's regimen? A) Diltiazem (Cardizem) B) Hydralazine C) Hydrochlorothiazide D) Metoprolol E) Olmesartan (Benicar)

E He is unable to tolerate ACE inhibitors, so he should begin taking an angiotensin receptor blocker (

A 55-year-old male sees you because of a second flare of gout. He has also had an elevated blood pressure at the last few visits to your clinic and is hypertensive again today. In addition to treating his gout flare, which one of the following would be the most appropriate agent to treat his hypertension in light of his presenting problem? A) Atenolol (Tenormin) B) Hydralazine C) Hydrochlorothiazide D) Lisinopril (Prinivil, Zestril) E) Losartan (Cozaar)

E Hydrochlorothiazide should typically be changed to another agent, such as losartan, when feasible in patients with gout. The American College of Rheumatology guideline does not recommend for or against the use of atenolol, hydralazine, and lisinopril as antihypertensive treatment in patients with gout.

A 61-year-old female comes to your office for a routine health maintenance visit. She brings in laboratory results from a local blood donation facility. She donated blood for the first time in several years but was informed afterward that she is ineligible and should follow up with her primary care physician. She feels well and is asymptomatic. She does not recall receiving the hepatitis B immunization series. The letter she received included the following hepatitis B test results: Anti-HBC screening test: positive ABsAg: Negative HBV NAT: Negative. Additional Hep B testing: Anti-HBs: 56 (N<10) Anti HBc IgM: nonreactive. Which one of the following is the most likely explanation of these laboratory results? A) These results indicate a false positive B) She was never infected with hepatitis B C) She has an acute hepatitis B infection D) She has a chronic hepatitis B infection E) She has recovered from a past hepatitis B infection

E If she had never been infected her anti-HBc and anti-HBs would both be negative. If she had an acute infection the HBsAg, anti-HBc IgM, and HBV nucleic acid test (NAT) would have all been positive along with the total anti-HBc. If she had a chronic infection the HBsAg and HBV NAT would be positive in addition to the total anti-HBc.

A 56-year-old female with a history of stage III non-small cell lung cancer who is currently receiving radiation treatment and chemotherapy sees you because of a poor appetite and a 4.5-kg (10-lb) weight loss in the past month. She requests medication to improve her appetite and you consider prescribing megestrol (Megace). Which one of the following is a possible side effect associated with the use of megestrol in this patient? A) Hirsutism B) Hypoglycemia C) Improved libido D) Thrombocytopenia E) A venous thromboembolic event

E Megestrol increases the risk of venous thromboembolic events in patients with cancer who are receiving chemotherapy (SOR C). Megestrol can also cause adrenal suppression, diabetes mellitus, and cardiomyopathy, and it is associated with alopecia, hyperglycemia, decreased libido, and sexual dysfunction.

A 63-year-old male presents for advice on smoking cessation. He was recently hospitalized with acute coronary syndrome and is highly motivated to quit smoking. He says that he has quit several times in the past but never for an extended period of time. He currently smokes 10 cigarettes per day. In addition to behavioral support resources such as 1-800-QUIT-NOW, you decide to prescribe a nicotinic receptor partial agonist. Which one of the following belongs to this class of medications? A) Acamprosate B) Bupropion (Wellbutrin SR) C) Clonidine (Catapres) D) Naltrexone E) Varenicline (Chantix)

E Memorization question Acamprosate is a gamma-aminobutyric acid (GABA) agonist and glutamate antagonist that is effective for the treatment of alcohol use disorder. Bupropion is a norepinephrine-dopamine reuptake inhibitor that reduces nicotine withdrawal and the reward from tobacco smoking. Clonidine is an 2-adrenergic agonist, and it has been shown to assist with smoking cessation but is not FDA approved for this purpose. Naltrexone is a pure opioid receptor antagonist that is effective for the treatment of alcohol use disorder and opioid use disorder.

A 78-year-old male with terminal lung cancer and long-standing COPD is admitted to a regular medical-surgical care unit pending transfer to the hospice unit within the next day. You are called about worsening anxiety and dyspnea. The patient is alert and anxious. He has a blood pressure of 150/94 mm Hg, a pulse rate of 96 beats/min, a respiratory rate of 24/min, and an oxygen saturation of 93% on 2 L/min of oxygen via nasal cannula. Which one of the following would be most effective in this situation? A) 40% oxygen by venti-mask B) Dexamethasone C) Hyoscyamine (Anaspaz) D) Lorazepam (Ativan) E) Morphine sulfate

E Opiates are the most effective agents for treating dyspnea and the resultant anxiety in patients with terminal cancer. Higher levels of oxygen are indicated if the patient's oxygen saturation is <92% and with caution in patients with COPD so as not to suppress respiratory drive. Dexamethasone, hyoscyamine, and lorazepam have a frequent role in patients such as this one, but morphine sulfate or a similar fast-acting opiate is the drug of choice (SOR B).

A 27-year-old soccer player presents with anterior hip pain along with a clicking sensation in the hip when he runs or attempts lateral movements. The flexion, adduction, and internal rotation (FADIR) test and the flexion, abduction, and external rotation (FABER) test both elicit pain. You suspect a labral tear. The most accurate imaging test for the suspected diagnosis is A) a standing radiograph of the hip B) anteroposterior and frog-leg lateral radiographs of the hip C) CT of the hip D) a bone scan E) MR arthrography

E Patients with labral tears usually present with anterior hip pain and may have catching, popping, or clicking sounds associated with activities such as gymnastics, soccer, dancing, basketball, or hockey. On physical examination the flexion, adduction, and internal rotation (FADIR) test and the flexion, abduction, and external rotation (FABER) test will elicit pain. Although initial imaging may include radiographs of the hip, MRI is often needed for diagnosis. MR arthrography with gadolinium injection into the hip joint has been the standard to diagnose labral tears.

Which one of the following has been shown to be an appropriate therapeutic intervention for nonspecific low back pain? A) Bed rest B) A lumbar brace C) Muscle relaxants D) Shoe insoles E) Yoga

E Physical activity, including core strengthening, physical therapy, or yoga, is an important therapeutic intervention in the treatment of nonspecific low back pain.

A 15-year-old female is brought to your office for a routine wellness check. Her only concerns are that she has never menstruated and she is not growing as fast as her peers. She is very active and plays volleyball on a travel team. An examination reveals that her height is now at the 25th percentile, although it was at the 90th percentile when she was 8 years old. She has breast buds that do not extend beyond the areola and her pubic hair is fine and sparse. Laboratory findings include a negative pregnancy test and a normal CBC, metabolic panel, TSH level, and prolactin level. She has an estradiol level of 12 pg/mL (N 25-75), an LH level of 40 mIU/mL (N 5-20), and an FSH level of 50 mIU/mL (N 3-20). The most likely diagnosis is A) congenital adrenal hyperplasia B) constitutional delay of puberty C) pituitary adenoma D) polycystic ovary syndrome E) primary ovarian insufficiency

E Primary amenorrhea is the lifelong absence of menses. If menarche has not occurred by age 15, or no menses have occurred 3 years after the development of breast buds, an evaluation is recommended. Primary ovarian insufficiency is associated with low estradiol levels and high levels of LH and FSH. Generally, the LH/FSH ratio is <1. Patients with congenital adrenal hyperplasia will have low estrogen, LH, and FSH levels. Virilization is generally noted in congenital adrenal hyperplasia, and a 17-hydroxyprogesterone level should be obtained to assess for this condition. Functional hypothalamic amenorrhea will also cause low levels of LH, FSH, and TSH. While polycystic ovary syndrome is associated with low estrogen, LH, and FSH levels, prolactin may be elevated. A pituitary adenoma will cause the prolactin level to be elevated.

A 72-year-old female with a history of type 2 diabetes presents with a 4-month history of a burning sensation along her tongue. Her diabetes is currently well controlled with metformin (Glucophage). Her other medications include atorvastatin (Lipitor) and lisinopril (Prinivil, Zestril). Her past medical history is otherwise unremarkable. An examination reveals a smooth, glossy, erythematous tongue (shown below). A laboratory evaluation should include which one of the following tests? A) Angiotensin converting enzyme B) Antineutrophil cytoplasmic antibody C) Serotonin D) TSH E) Vitamin B12

E Routine vitamin B12 monitoring or supplementation is appropriate for patients who take metformin chronically. A decline in vitamin B12 levels may be seen as early as 3-4 months after starting metformin.

A 45-year-old female presents with a lesion on her mid-back that measures 1.2 mm in diameter. A punch biopsy confirms nodular basal cell carcinoma. She is otherwise healthy and does not take any daily medications. She is concerned about the cosmetic appearance after treatment. Which one of the following would be the most appropriate treatment strategy? A) No further management B) Topical fluorouracil 5% (Efudex) C) Cryotherapy D) Curettage and electrodesiccation E) Standard excision with 4-mm margins

E Surgical excision is indicated for the management of larger basal cell carcinomas. Although this patient had a punch biopsy, that is not considered curative and excision with wide margins is indicated. Topical therapy and cryotherapy are reserved for patients who decline surgery or for cases in which surgery is contraindicated.

A 52-year-old female with no history of cigarette smoking is admitted to the hospital because of chest pain and shortness of breath. After an extensive evaluation you make a diagnosis of Takotsubo syndrome. Findings are most likely to be normal on which one of the following with this diagnosis? A) A CK-MB level B) A troponin level C) An EKG D) Echocardiography E) Coronary angiography

E Takotsubo syndrome (acute stress-induced cardiomyopathy) is characterized by transient wall motion abnormalities on echocardiography, usually following an emotionally triggering event. Although coronary artery disease may be present, and the two diagnoses may be seen together, the International Takotsubo Registry reports coronary artery disease in only about 15%, so coronary angiography is more likely to be normal than the other listed diagnostic tests.

A patient begins to cry when you tell her that the mammogram she had yesterday shows an abnormality requiring further imaging. The most appropriate response at this time is to A) tell her there is no need to cry B) quickly reassure her that this is most likely benign C) reassure her that most breast cancers have a long survival rate D) wait to specifically address her emotional response until after you have given her more complete information E) stop giving information and address her apparent sadness prior to continuing

E The physician should accept the patient's response and acknowledge it at that time, most appropriately with a statement that shows empathy for the emotion. This should be done prior to attempting to immediately reassure her about the prognosis or giving more information.

During a quality improvement project you notice significant variations in the ordering patterns of physicians in your group when checking for proteinuria. Some physicians routinely order spot protein/creatinine ratios while others order spot albumin/creatinine ratios. When comparing these two options, one advantage of spot albumin/creatinine ratios is that they are A) more convenient for the patient B) unaffected by exercise C) unaffected by menstruation D) elevated only in patients with diabetes mellitus E) able to detect lower levels of proteinuria

E The spot albumin/creatinine ratio is able to detect lower levels of proteinuria as compared to the spot protein/creatinine ratio. The albumin/creatinine ratio indicates proteinuria, which is not specific to diabetes mellitus. Both protein/creatinine and albumin/creatinine ratios can be affected by exercise and menstruation

To determine compliance with prescribed medications and detect use of illicit substances, your clinic uses urine drug screening with an immunoassay qualitative point-of-care test to monitor patients who are on long-term opioid therapy. Which one of the following is most likely to result in a false-negative result and require confirmatory testing for detection? A) Cannabis B) Cocaine C) Codeine D) Morphine E) Oxycodone (OxyContin)

E These immunoassays do not reliably detect synthetic or semisynthetic opioids such as oxycodone, oxymorphone, methadone, buprenorphine, and fentanyl, as well as many benzodiazepines.

A 45-year-old male presents to your office with intermittent chest pain for the past few days, although he is currently pain free after taking aspirin at home. He tells you that while running this morning he had pain every time he ran uphill. The pain is a dull ache on his left chest wall. He has no other associated symptoms and no significant past medical history or family history. His vital signs are stable and a physical examination is unremarkable. EKG shows new LBBB. Which one of the following would be most appropriate at this point? A) An exercise stress test B) Stress echocardiography C) Coronary CT angiography D) Referral to a cardiologist E) Transportation via ambulance to the emergency department

E This patient presents with risk factors for coronary artery disease, including male sex and activity-related chest pain. He also has a new left bundle branch block, which necessitates a trip to the emergency department for urgent evaluation

A 42-year-old female who owns a bakery presents with a several-month history of gradually worsening pain, swelling, and paresthesia affecting her entire right arm whenever she has to blend ingredients by hand. She says that her "arm veins pop out" and her arm develops a deep aching pain if she has to stir mixes for very long. The pain and swelling have become so severe that she is no longer able to make wedding cakes and is concerned she will be unable to continue running her business. She does not recall any trauma and has no swelling in her left arm. She has a history of essential hypertension that is treated with losartan (Cozaar). On examination the patient has full active range of motion and the Neer and Hawkins impingement tests of the shoulder are negative. Based on this patient's history and the physical examination findings, which one of the following is the most likely diagnosis? A) Carpal tunnel syndrome B) Cervical radiculopathy C) Complex regional pain syndrome D) Raynaud's disease E) Thoracic outlet syndrome

E Thoracic outlet syndrome can be differentiated into neurogenic, venous, or arterial, with neurogenic being the most common, constituting more than 95% of cases. This patient has venous thoracic outlet syndrome, which is the second most common, occurring in about 3% of cases. Swelling of the arm with associated pain strongly suggests obstruction of the subclavian vein. Paresthesias in the fingers and hand are common, likely due to swelling rather than nerve compression at the thoracic outlet. High risk of developing venous thrombosis

A 60-year-old female with a BMI of 24 kg/m2 presents with bilateral knee pain that is greater in the left knee. She has no morning stiffness but the pain sometimes prevents her from completing normal activities such as grocery shopping. You perform an evaluation and diagnose osteoarthritis of the knee. She does not want to take prescription medications and asks you if supplements or other treatments would be helpful. Which one of the following would you recommend? A) Lateral wedge insoles B) Vitamin D supplements C) Glucosamine and chondroitin supplements D D) Hyaluronic acid injections E) Physical therapy

E Treatments with evidence of effectiveness for knee osteoarthritis include exercise, physical therapy, knee taping, and tai chi. Medical treatments should begin with full-strength acetaminophen and topical therapy, then NSAIDs and, selectively, tramadol or other opioid Per AAOS, glucosamine and chondroitin should not be used in knee osteoarthritis and lateral wedge insoles should not be used for medial knee osteoarthritis.

The U.S. Preventive Services Task Force currently recommends hepatitis C screening for A) high-risk adults only, regardless of age B) only high-risk adults born between 1945 and 1965 C) adults born between 1945 and 1965 regardless of risk, and other adults only if they are at high risk D) adults born between 1945 and 1980 regardless of risk, and other adults only if they are at high risk E) all adults <80 years of age, regardless of risk

E USPSTF currently recommends that all asymptomatic adults, including pregnant women, between the ages of 18 and 79 without known liver disease should be screened for hepatitis C virus (B recommendation).

A 30-year-old male comes to your office for evaluation of hand weakness. On examination you detect weakness when he tries to bring his thumb and index finger together. For confirmation you ask him to try to hold on to a piece of paper between his thumb and index finger while you try to pull it away. He is unable to resist when you pull on the paper. The most likely explanation for these findings is an injury to the A) brachial plexus B) median nerve C) musculocutaneous nerve D) radial nerve E) ulnar nerve

E Weakness of the thumb and index finger pincer mechanism is indicative of an ulnar nerve injury. Weakness in the shoulder or upper arm would indicate a potential brachial plexus injury.

A 29-year-old male tells you that several years ago he was physically assaulted while walking home from work. Since the assault he has experienced insomnia, anhedonia, irritability, and vivid flashbacks and intrusive thoughts about the assault. Using a screening tool and structured interview you make a diagnosis and discuss treatment. Which one of the following would be the best evidence-based recommendation for initial treatment? A) Alprazolam (Xanax) B) Clonazepam (Klonopin) C) Escitalopram (Lexapro) D) Dialectical behavioral therapy E) Individual trauma-focused psychotherapy

E individual trauma-focused psychotherapy is the intervention that demonstrates the most significant benefit. Pharmacotherapy may be used if psychotherapy is not effective or available. Recommended options include fluoxetine, paroxetine, venlafaxine, or sertraline.

A 42-year-old female presents to your office with heavy menstrual periods and pelvic pressure. Her symptoms began several years ago and have gradually worsened. Laboratory findings are notable for a mild microcytic anemia. Pelvic ultrasonography identifies a 7-cm submucosal mass. She wants to avoid a hysterectomy but desires a treatment that will provide symptom relief, decrease the volume of the mass, and have a sustained effect. Which one of the following would be most appropriate for this patient? A) Expectant management B) A GnRH agonist C) A selective estrogen receptor modulator D) A levonorgestrel-releasing IUD (Mirena) E) Uterine artery embolization and occlusion

E uterine artery embolization and occlusion is effective for reducing fibroid size, with lasting effects up to 5 years and moderate evidence for reducing bleeding and improving quality of life. Expectant management is an appropriate option only for patients who have asymptomatic fibroids.

You see a 55-year-old female for preoperative clearance prior to a cholecystectomy. When you examine the patient she asks you to also look at the lesion on her foot (shown below). Which one of the following is the most likely etiology of the lesion? A) Arterial B) Venous C) Infectious D) Neuropathic E) Pressure

The likely etiology of this patient's lesion is neuropathy, most likely due to poorly controlled diabetes mellitus. Peripheral neuropathy can predispose patients to abnormal gait patterns and/or unrecognized trauma. These deep ulcers usually present over a bony prominence and are surrounded by a callus


Ensembles d'études connexes

Module 4.1: Hypothesis Testing and Null Hypothesis Significance Testing

View Set

Abnormal Psych Chapter 1 Questions

View Set

Common Fractions with Decimal and Percent Equivalents

View Set

Electrical - J2 - UNVERIFIED QUESTIONS

View Set

KNR 380, Human Resources, Exam 1 Study Guide

View Set